Study Mode
Quiz-summary
0 of 151 questions completed
Questions:
- 1
- 2
- 3
- 4
- 5
- 6
- 7
- 8
- 9
- 10
- 11
- 12
- 13
- 14
- 15
- 16
- 17
- 18
- 19
- 20
- 21
- 22
- 23
- 24
- 25
- 26
- 27
- 28
- 29
- 30
- 31
- 32
- 33
- 34
- 35
- 36
- 37
- 38
- 39
- 40
- 41
- 42
- 43
- 44
- 45
- 46
- 47
- 48
- 49
- 50
- 51
- 52
- 53
- 54
- 55
- 56
- 57
- 58
- 59
- 60
- 61
- 62
- 63
- 64
- 65
- 66
- 67
- 68
- 69
- 70
- 71
- 72
- 73
- 74
- 75
- 76
- 77
- 78
- 79
- 80
- 81
- 82
- 83
- 84
- 85
- 86
- 87
- 88
- 89
- 90
- 91
- 92
- 93
- 94
- 95
- 96
- 97
- 98
- 99
- 100
- 101
- 102
- 103
- 104
- 105
- 106
- 107
- 108
- 109
- 110
- 111
- 112
- 113
- 114
- 115
- 116
- 117
- 118
- 119
- 120
- 121
- 122
- 123
- 124
- 125
- 126
- 127
- 128
- 129
- 130
- 131
- 132
- 133
- 134
- 135
- 136
- 137
- 138
- 139
- 140
- 141
- 142
- 143
- 144
- 145
- 146
- 147
- 148
- 149
- 150
- 151
Information
Study Mode
You have already completed the quiz before. Hence you can not start it again.
Quiz is loading...
You must sign in or sign up to start the quiz.
You have to finish following quiz, to start this quiz:
Results
0 of 151 questions answered correctly
Your time:
Time has elapsed
You have reached 0 of 0 points, (0)
Categories
- Community Heath & Psychiatry 0%
- Obstetrics & Gynecology 0%
- Study Mode 0%
- Surgery 0%
- 1
- 2
- 3
- 4
- 5
- 6
- 7
- 8
- 9
- 10
- 11
- 12
- 13
- 14
- 15
- 16
- 17
- 18
- 19
- 20
- 21
- 22
- 23
- 24
- 25
- 26
- 27
- 28
- 29
- 30
- 31
- 32
- 33
- 34
- 35
- 36
- 37
- 38
- 39
- 40
- 41
- 42
- 43
- 44
- 45
- 46
- 47
- 48
- 49
- 50
- 51
- 52
- 53
- 54
- 55
- 56
- 57
- 58
- 59
- 60
- 61
- 62
- 63
- 64
- 65
- 66
- 67
- 68
- 69
- 70
- 71
- 72
- 73
- 74
- 75
- 76
- 77
- 78
- 79
- 80
- 81
- 82
- 83
- 84
- 85
- 86
- 87
- 88
- 89
- 90
- 91
- 92
- 93
- 94
- 95
- 96
- 97
- 98
- 99
- 100
- 101
- 102
- 103
- 104
- 105
- 106
- 107
- 108
- 109
- 110
- 111
- 112
- 113
- 114
- 115
- 116
- 117
- 118
- 119
- 120
- 121
- 122
- 123
- 124
- 125
- 126
- 127
- 128
- 129
- 130
- 131
- 132
- 133
- 134
- 135
- 136
- 137
- 138
- 139
- 140
- 141
- 142
- 143
- 144
- 145
- 146
- 147
- 148
- 149
- 150
- 151
- Answered
- Review
-
Question 1 of 151
1. Question
25-year-old man has a several year history of occasionally noting pink discoloration in his morning urine. He presents to the emergency room three days after developing a severe cold with symptoms of abdominal and lumbar pain. A urine sample appears grossly bloody. No cells are seen when a urine smear is made, but hemosiderin crystals are noted. The hematocrit is 25%, and the overlying serum is dark pink in color. The red cell morphology on peripheral smear is normal. Which of the following tests is most likely to confirm the diagnosis?
Correct
Correct answer is A. This patient’s probable disease is paroxysmal nocturnal hemoglobinuria, also known as Marchiafava-Micheli syndrome. This rare disorder appears to have a genetic predisposition related to the PIG-A gene on the X-chromosome that makes a glycosyl-phosphatidyl-inositol “anchor” used by a variety of membrane proteins. Paroxysmal nocturnal hemoglobinuria behaves like an acquired hemolytic anemia with marked sensitivity to serum complement factor C3. Affected patients are most often men in their 20’s, but both sexes
and all ages can develop the condition.Incorrect
Correct answer is A. This patient’s probable disease is paroxysmal nocturnal hemoglobinuria, also known as Marchiafava-Micheli syndrome. This rare disorder appears to have a genetic predisposition related to the PIG-A gene on the X-chromosome that makes a glycosyl-phosphatidyl-inositol “anchor” used by a variety of membrane proteins. Paroxysmal nocturnal hemoglobinuria behaves like an acquired hemolytic anemia with marked sensitivity to serum complement factor C3. Affected patients are most often men in their 20’s, but both sexes
and all ages can develop the condition. -
Question 2 of 151
2. Question
35-year-old male with end-stage renal disease presents with syncope. The patient denies chest pain or dyspnea. Physical examination is remarkable for a left extremity AV fistula. His creatinine is 14 mg/dL, urea nitrogen is 88 mg/dL and potassium is 8.8 mEq/L. ECG reveals sinus bradycardia with a sine wave pattern. Which of the following is he most appropriate initial step in management?
Correct
The correct answer is A. IV calcium will work very rapidly to counteract the effect of the high potassium on the heart and muscle and should be given first. The calcium will not actually lower the level of potassium, however. All of the other agents are good at lowering potassium, but do not have the same immediate effect to spare the muscle that calcium has. Therefore, these agents should be given after IV calcium when there are severe ECG changes.
Incorrect
The correct answer is A. IV calcium will work very rapidly to counteract the effect of the high potassium on the heart and muscle and should be given first. The calcium will not actually lower the level of potassium, however. All of the other agents are good at lowering potassium, but do not have the same immediate effect to spare the muscle that calcium has. Therefore, these agents should be given after IV calcium when there are severe ECG changes.
-
Question 3 of 151
3. Question
A 70-year-old woman is brought to the physician by her daughter because of personality change and forgetfulness for the past 3 months. Her last medical check-up was 1 year earlier, when she appeared in good spirits and excellent health. Six months ago, her husband died. During the examination, the patient answers the physician’s questions with “Yes,” “No,” or “I don’t know” and appears apathetic and indifferent. When asked questions testing short-term memory, she often replies “I don’t know.” Physical examination is unremarkable. Blood studies are normal, and TSH is within normal range. The physician arranges for an MRI examination of the brain. The T1-weighted image is shown below. Which of the following is the most likely diagnosis?
Correct
The correct answer is E. The patient’s MRI scan does not reveal any significant cortical atrophy. Cortical atrophy manifests with narrowing of gyri and widening of sulci. As a secondary effect, enlargement of the ventricles takes place, a feature known as hydrocephalus ex vacuo. The ventricles are normal in this case. The patient’s behavior, lack of interest, and verbal replies suggest a diagnosis of depression. Elderly people with this condition often present with cognitive deficits, which may constitute the predominant clinical manifestation. Depressed patients tend to reply “I don’t know” to questions aimed at testing memory skills. A diagnosis of pseudodementia is applicable whenever severe cognitive deficits are not explained by objective findings by clinical, radiologic, or pathologic investigations. Depression is certainly an important cause to consider in the differential diagnosis of dementing disorders.
Alzheimer disease (choice A) and Pick dementia (choice D) are excluded not only by the clinical picture but also by the lack of cortical atrophy on MRI. Whereas Alzheimer dementia is characterized by diffuse cortical atrophy, Pick disease has a specific predilection for the frontal and temporal lobes.
Brain tumor (choice B) may be ruled out by the neuroimaging studies. Although brain tumors may manifest with slowly progressive memory deficits mimicking dementia (e.g., meningiomas), they are usually associated with focal neurologic signs and symptoms. Normal pressure hydrocephalus (choice C) is a rare form of treatable dementing illness due to recurrent transient increases in CSF pressure, with resultant progressive enlargement of ventricles. The ventricular cavities appear entirely within normal limits in this case.
Incorrect
The correct answer is E. The patient’s MRI scan does not reveal any significant cortical atrophy. Cortical atrophy manifests with narrowing of gyri and widening of sulci. As a secondary effect, enlargement of the ventricles takes place, a feature known as hydrocephalus ex vacuo. The ventricles are normal in this case. The patient’s behavior, lack of interest, and verbal replies suggest a diagnosis of depression. Elderly people with this condition often present with cognitive deficits, which may constitute the predominant clinical manifestation. Depressed patients tend to reply “I don’t know” to questions aimed at testing memory skills. A diagnosis of pseudodementia is applicable whenever severe cognitive deficits are not explained by objective findings by clinical, radiologic, or pathologic investigations. Depression is certainly an important cause to consider in the differential diagnosis of dementing disorders.
Alzheimer disease (choice A) and Pick dementia (choice D) are excluded not only by the clinical picture but also by the lack of cortical atrophy on MRI. Whereas Alzheimer dementia is characterized by diffuse cortical atrophy, Pick disease has a specific predilection for the frontal and temporal lobes.
Brain tumor (choice B) may be ruled out by the neuroimaging studies. Although brain tumors may manifest with slowly progressive memory deficits mimicking dementia (e.g., meningiomas), they are usually associated with focal neurologic signs and symptoms. Normal pressure hydrocephalus (choice C) is a rare form of treatable dementing illness due to recurrent transient increases in CSF pressure, with resultant progressive enlargement of ventricles. The ventricular cavities appear entirely within normal limits in this case.
-
Question 4 of 151
4. Question
A middle aged man develops a scleroderma-like illness with thickening of the skin. The thickened skin is most marked over the anterior surfaces of the extremities and has a characteristic “orange peel” configuration. The man has not experienced Raynaud’s phenomenon.
No telangiectasias and no calcinosis is seen. These skin changes should suggest which of the following diagnoses?Correct
The correct answer is C. Orange peel skin on the anterior aspects of the extremities is a distinctive feature of eosinophilic fasciitis, which is a scleroderma-like disorder involving the arms, legs, and sometimes face and trunk, but not usually the hands and feet. Middle aged men are most commonly affected. Symptoms usually develop insidiously, and lead to eventual restriction of arm and leg motion. The restriction of movement is related to inflammation and fibrosis of fascia and sometimes tendons, synovial membranes, and muscle. Biopsy of the skin or fascia shows cellular infiltrates with histiocytes, plasma cells, lymphocytes, and (in only some cases, despite the name) eosinophils. Treatment is usually with initially high dose prednisone followed by tapering and maintenance for 2 to 5 years on low dose prednisone.
CREST syndrome (choice A) is a variant ofscleroderma with prominent involvement of the hands (sclerodactyly), calcinosis, Raynaud’s phenomenon, telangiectasias, and esophageal dysfunction. Dermatomyositis (choice B) produces inflammatory and degenerative changes in muscles and skin. Skin manifestations include a dusky, erythematous rash, which may show desquamation with splitting of the skin, particularly of the fingers.
Polymyalgia rheumatica (choice D) does not cause skin manifestations.
Polymyositis (choice E) by itself does not cause skin manifestations; if it is accompanied by dusky rash, it is called
dermatomyositis.Incorrect
The correct answer is C. Orange peel skin on the anterior aspects of the extremities is a distinctive feature of eosinophilic fasciitis, which is a scleroderma-like disorder involving the arms, legs, and sometimes face and trunk, but not usually the hands and feet. Middle aged men are most commonly affected. Symptoms usually develop insidiously, and lead to eventual restriction of arm and leg motion. The restriction of movement is related to inflammation and fibrosis of fascia and sometimes tendons, synovial membranes, and muscle. Biopsy of the skin or fascia shows cellular infiltrates with histiocytes, plasma cells, lymphocytes, and (in only some cases, despite the name) eosinophils. Treatment is usually with initially high dose prednisone followed by tapering and maintenance for 2 to 5 years on low dose prednisone.
CREST syndrome (choice A) is a variant ofscleroderma with prominent involvement of the hands (sclerodactyly), calcinosis, Raynaud’s phenomenon, telangiectasias, and esophageal dysfunction. Dermatomyositis (choice B) produces inflammatory and degenerative changes in muscles and skin. Skin manifestations include a dusky, erythematous rash, which may show desquamation with splitting of the skin, particularly of the fingers.
Polymyalgia rheumatica (choice D) does not cause skin manifestations.
Polymyositis (choice E) by itself does not cause skin manifestations; if it is accompanied by dusky rash, it is called
dermatomyositis. -
Question 5 of 151
5. Question
A 9-year-old boy is being evaluated for a syncopal episode. He was playing basketball when the syncope occurred. The episode lasted about 1-2 minutes, and then the patient woke up on his own. His mother also noticed that he has been having hearing problems. His physical examination is unremarkable. An ECG shows a prolonged Q-T interval, with a QTc of 0.49 sec. Which of the following is the most appropriate management if he has long Q-T syndrome.
Correct
The correct answer is C. The boy in this clinical vignette has long Q-T syndrome (LQTS). About 50% of cases are familial: Romano-Ward syndrome has autosomal-dominant transmission; Jervell-Lange-Nielsen syndrome has autosomal-recessive transmission. LQTS occurs in all races and ethnic groups. The principal symptoms are syncope and sudden death from torsades de pointes (TDP). Most often, TDP is self-terminating and causes a syncopal episode from which the patient quickly recovers. Cardiac arrest occurs if the TDP is more persistent, and sudden death results if the rhythm does not return to normal spontaneously and the patient is not resuscitated.
Syncope, caused by TDP, is the primary symptom in inherited LQTS. Patients may have one to hundreds of episodes. The symptoms commonly occur within the first few years of life in patients with Jervell-Lange-Nielsen syndrome, and the mortality rate with this form is higher than in patients with Romano-Ward. In Romano-Ward syndrome, the median ages at symptom onset and sudden death are in the pre- to mid-teenage years. Of interest, at least one third of gene carriers never develop symptoms, lead completely normal lifestyles, and have normal life spans. Approximately one third has just one to a few syncopal spells as children, then none thereafter. Syncope and sudden death occur most often during exercise or intense emotion, with an important minority occurring during sleep. Events are uncommon while patients are awake and at rest and without apparent provocation. The predominant feature on an ECG is QT prolongation. The QTc averages 0. 49 seconds.
The gold-standard therapy for LQTS remains beta-blocker administration, which is effective in 80% to 90% of patients, with a significant reduction in the rate of sudden death. Asymptomatic children and young persons should be treated prophylactically with beta-blockers on diagnosis. The implantable cardiac defibrillator (ICD) is being used with increasing frequency, especially in apparently high-risk patients, such as those experiencing TDP while on beta-blockers and those who have had cardiac arrest.
Incorrect
The correct answer is C. The boy in this clinical vignette has long Q-T syndrome (LQTS). About 50% of cases are familial: Romano-Ward syndrome has autosomal-dominant transmission; Jervell-Lange-Nielsen syndrome has autosomal-recessive transmission. LQTS occurs in all races and ethnic groups. The principal symptoms are syncope and sudden death from torsades de pointes (TDP). Most often, TDP is self-terminating and causes a syncopal episode from which the patient quickly recovers. Cardiac arrest occurs if the TDP is more persistent, and sudden death results if the rhythm does not return to normal spontaneously and the patient is not resuscitated.
Syncope, caused by TDP, is the primary symptom in inherited LQTS. Patients may have one to hundreds of episodes. The symptoms commonly occur within the first few years of life in patients with Jervell-Lange-Nielsen syndrome, and the mortality rate with this form is higher than in patients with Romano-Ward. In Romano-Ward syndrome, the median ages at symptom onset and sudden death are in the pre- to mid-teenage years. Of interest, at least one third of gene carriers never develop symptoms, lead completely normal lifestyles, and have normal life spans. Approximately one third has just one to a few syncopal spells as children, then none thereafter. Syncope and sudden death occur most often during exercise or intense emotion, with an important minority occurring during sleep. Events are uncommon while patients are awake and at rest and without apparent provocation. The predominant feature on an ECG is QT prolongation. The QTc averages 0. 49 seconds.
The gold-standard therapy for LQTS remains beta-blocker administration, which is effective in 80% to 90% of patients, with a significant reduction in the rate of sudden death. Asymptomatic children and young persons should be treated prophylactically with beta-blockers on diagnosis. The implantable cardiac defibrillator (ICD) is being used with increasing frequency, especially in apparently high-risk patients, such as those experiencing TDP while on beta-blockers and those who have had cardiac arrest.
-
Question 6 of 151
6. Question
5-year-old boy suffers from a condition characterized by recurrent fungal and viral infections, thymic hypoplasia, tetany, and abnormal facies. Serum levels of immunoglobulins are mildly depressed, and lymph node biopsy shows lymphocyte depletion of T-dependent areas. Which of the following is the underlying pathogenetic mechanism?
Correct
The correct answer is A. The constellation of thymic hypoplasia, hypocalcemia (with tetany), abnormal facies, and congenital
cardiac anomalies defines the condition known as DiGeorge syndrome. This results from a developmental failure of third and
fourth pharyngeal pouches, which gives rise to congenital absence or anomalies of the parathyroid, thymus, lower face, and cardiac
structures. Immune deficiency results from failure of T-lymphocytes to mature in the thymus. Thus, fungal and viral organisms, which are normally controlled by T-mediated mechanisms, become frequent causes of opportunistic infections. The underlying gene defect is related to 22q11 deletion, which results in two partially overlapping conditions, i.e. DiGeorge syndrome and velocardiofacial syndrome. These conditions are collectively referred to as chromosome 22q11 deletion syndrome.In utero infection by human immunodeficiency virus (HIV) (choice B) would result in T-cell deficiency but would not be associated with congenital abnormalities.
Mutations of the autosomal gene encoding adenosine deaminase (choice C) represent the most common cause of the recessive form of severe combined immunodeficiency disease (SCID), encompassing a heterogeneous group of conditions characterized by deficiency of both T- and B-cell mechanisms. SCID may be autosomal dominant, autosomal recessive, or X-linked.
Mutations of the X-linked gene coding for a cytokine receptor subunit (choice D) represent the most common cause of the autosomal dominant form of SCID.
Mutation of an X-linked gene coding for a tyrosine kinase (choice E) is the underlying molecular mechanism leading to X-linked agammaglobulinemia of Bruton, a syndrome characterized by inability of pre-B cell precursors to mature into B-lymphocytes. Humoral immune deficiency thus manifests.
Incorrect
The correct answer is A. The constellation of thymic hypoplasia, hypocalcemia (with tetany), abnormal facies, and congenital
cardiac anomalies defines the condition known as DiGeorge syndrome. This results from a developmental failure of third and
fourth pharyngeal pouches, which gives rise to congenital absence or anomalies of the parathyroid, thymus, lower face, and cardiac
structures. Immune deficiency results from failure of T-lymphocytes to mature in the thymus. Thus, fungal and viral organisms, which are normally controlled by T-mediated mechanisms, become frequent causes of opportunistic infections. The underlying gene defect is related to 22q11 deletion, which results in two partially overlapping conditions, i.e. DiGeorge syndrome and velocardiofacial syndrome. These conditions are collectively referred to as chromosome 22q11 deletion syndrome.In utero infection by human immunodeficiency virus (HIV) (choice B) would result in T-cell deficiency but would not be associated with congenital abnormalities.
Mutations of the autosomal gene encoding adenosine deaminase (choice C) represent the most common cause of the recessive form of severe combined immunodeficiency disease (SCID), encompassing a heterogeneous group of conditions characterized by deficiency of both T- and B-cell mechanisms. SCID may be autosomal dominant, autosomal recessive, or X-linked.
Mutations of the X-linked gene coding for a cytokine receptor subunit (choice D) represent the most common cause of the autosomal dominant form of SCID.
Mutation of an X-linked gene coding for a tyrosine kinase (choice E) is the underlying molecular mechanism leading to X-linked agammaglobulinemia of Bruton, a syndrome characterized by inability of pre-B cell precursors to mature into B-lymphocytes. Humoral immune deficiency thus manifests.
-
Question 7 of 151
7. Question
A 4-week-old boy is brought to clinic by his mother because of a 1 day history of labored breathing. His birth was uneventful and immunizations have been up to date. His mother reports that the patient developed conjunctivitis on the fourth day of life. On physical examination, he is breathing rapidly at 40 breaths per minute and is afebrile. His chest reveals bilateral inspiratory crackles and a slight wheeze. On chest x-ray, bilateral pneumonia is evident. The leukocyte count is elevated at 15,000 with 40% eosinophils. Which of the following is the most likely pathogen causing the patient’s symptoms?
Correct
The correct answer is B. This patient presents with symptoms consistent with neonatal pneumonia due to Chlamydia trachomatis.
This agent is transmitted from the mother’s vaginal secretions to the neonate at birth. The conjunctivitis precedes the pneumonitis.
Tachypnea, hypoxemia, crackles, wheezing and eosinophilia are seen.Ascaris lumbricoides(choice A) produce visceral larva migrans and can cause pneumonia and eosinophilia. However, this is caused by
ingestion of Ascaris eggs passed by dogs and cats.Mycoplasma pneumoniae(choice C) is the etiologic agent in atypical pneumonias in young people, but not in neonates. Pneumocystis carinii(choice D) causes pneumonia in patients with acquired immune deficiency syndrome and other immunocompromised patients.
Varicella(choice E) pneumonia is accompanied by skin lesions. Furthermore, eosinophilia is not seen.
Incorrect
The correct answer is B. This patient presents with symptoms consistent with neonatal pneumonia due to Chlamydia trachomatis.
This agent is transmitted from the mother’s vaginal secretions to the neonate at birth. The conjunctivitis precedes the pneumonitis.
Tachypnea, hypoxemia, crackles, wheezing and eosinophilia are seen.Ascaris lumbricoides(choice A) produce visceral larva migrans and can cause pneumonia and eosinophilia. However, this is caused by
ingestion of Ascaris eggs passed by dogs and cats.Mycoplasma pneumoniae(choice C) is the etiologic agent in atypical pneumonias in young people, but not in neonates. Pneumocystis carinii(choice D) causes pneumonia in patients with acquired immune deficiency syndrome and other immunocompromised patients.
Varicella(choice E) pneumonia is accompanied by skin lesions. Furthermore, eosinophilia is not seen.
-
Question 8 of 151
8. Question
A 18-year-old man presents for a health maintenance examination. He has a family history of Tangier disease, and a number of adults in his family have either hepatosplenomegaly, recurrent polyneuropathy, or both. Which of the following would be the strongest finding on physical examination to suggest the presence of this disease?
Correct
The correct answer is D. Tangier disease is a rare familial disorder characterized by alpha-lipoprotein deficiency, which leads to very low high-density lipoprotein (HDL), recurrent polyneuropathy, lymphadenopathy, and hepatosplenomegaly due to storage of cholesterol esters in reticuloendothelial cells. Although you may never see this disease, the association of orange-yellow tonsillar hyperplasia (due to the cholesterol ester deposition there as well) with Tangier disease is a sufficiently distinctive clue in physical diagnosis to be worth remembering. (One rule of thumb in medicine is that although you will probably never see most of the very rare diseases, you will almost certainly see some of them.)
The presence of multiple angiokeratomas (choice A) on the lower half of the body suggests Fabry disease.
Grey-brown pigmentation of the forehead, hands, and pre-tibial region (choice B) suggests Gaucher disease.
Irregular black deposits of clumped pigment in the peripheral retina (choice C) are characteristic of retinitis pigmentosa, which may occur in association with abetalipoproteinemia and Refsum disease.
Pingueculae (choice E) could suggest Gaucher disease, but this could also occur in normal adults.
Incorrect
The correct answer is D. Tangier disease is a rare familial disorder characterized by alpha-lipoprotein deficiency, which leads to very low high-density lipoprotein (HDL), recurrent polyneuropathy, lymphadenopathy, and hepatosplenomegaly due to storage of cholesterol esters in reticuloendothelial cells. Although you may never see this disease, the association of orange-yellow tonsillar hyperplasia (due to the cholesterol ester deposition there as well) with Tangier disease is a sufficiently distinctive clue in physical diagnosis to be worth remembering. (One rule of thumb in medicine is that although you will probably never see most of the very rare diseases, you will almost certainly see some of them.)
The presence of multiple angiokeratomas (choice A) on the lower half of the body suggests Fabry disease.
Grey-brown pigmentation of the forehead, hands, and pre-tibial region (choice B) suggests Gaucher disease.
Irregular black deposits of clumped pigment in the peripheral retina (choice C) are characteristic of retinitis pigmentosa, which may occur in association with abetalipoproteinemia and Refsum disease.
Pingueculae (choice E) could suggest Gaucher disease, but this could also occur in normal adults.
-
Question 9 of 151
9. Question
During a camping trip, a 12-year-old boy scout is bitten by a rattlesnake on his right ankle. Pain and swelling of the bitten area become rapidly intense. While the boy is transported to the closest medical facility, which of the following is the most appropriate initial step in management?
Correct
The correct answer is D. The most crucial measure is to transport the victim of a snakebite to the nearest medical facility for intravenous administration of antivenin horse serum. The most current recommendations for “field” first aid include immobilization of the patient and the bitten part in a horizontal position. Systemic spread of the venom can be retarded if the victim remains as inactive as possible. The bitten limb can be splinted and kept at heart level if feasible. The clinical manifestations depend on the type of venom, whether predominantly cytolytic (rattlesnake and other pit vipers) or neurotoxic (coral snake). Predominantly cytotoxic venom causes local pain, redness, swelling, followed by perioral paresthesias and metallic taste; sometimes, shock, and coagulopathy may develop. Neurotoxic envenomation (e.g. coral snake) causes signs and symptoms similar to botulism, i.e. diplopia, dysphagia, ptosis, and respiratory arrest.
Any manipulation of the bitten area, such as application of ice (choice A), tourniquet (choice B), or incision and suction (choice E) is strongly discouraged. These measures are either ineffective in preventing the venom from entering circulation or result in unnecessary local trauma and ischemia. Commercially available mechanical devices for local suction may be helpful if used within 5 minutes, but mouth suction results in insignificant venom extraction and may lead to contamination of the wound by oral bacteria.
Giving alcohol (choice C) or stimulants to victims of snakebites is also useless or harmful.
Incorrect
The correct answer is D. The most crucial measure is to transport the victim of a snakebite to the nearest medical facility for intravenous administration of antivenin horse serum. The most current recommendations for “field” first aid include immobilization of the patient and the bitten part in a horizontal position. Systemic spread of the venom can be retarded if the victim remains as inactive as possible. The bitten limb can be splinted and kept at heart level if feasible. The clinical manifestations depend on the type of venom, whether predominantly cytolytic (rattlesnake and other pit vipers) or neurotoxic (coral snake). Predominantly cytotoxic venom causes local pain, redness, swelling, followed by perioral paresthesias and metallic taste; sometimes, shock, and coagulopathy may develop. Neurotoxic envenomation (e.g. coral snake) causes signs and symptoms similar to botulism, i.e. diplopia, dysphagia, ptosis, and respiratory arrest.
Any manipulation of the bitten area, such as application of ice (choice A), tourniquet (choice B), or incision and suction (choice E) is strongly discouraged. These measures are either ineffective in preventing the venom from entering circulation or result in unnecessary local trauma and ischemia. Commercially available mechanical devices for local suction may be helpful if used within 5 minutes, but mouth suction results in insignificant venom extraction and may lead to contamination of the wound by oral bacteria.
Giving alcohol (choice C) or stimulants to victims of snakebites is also useless or harmful.
-
Question 10 of 151
10. Question
A 43-year-old man reports that he has had a 9-pound weight loss over the past 9 months. The symptoms have been accompanied by difficulty swallowing both solids and liquids during that time. He has woken on several occasions at approximately 4 AM and regurgitated partially digested dinner contents. An upper gastrointestinal series is performed and reveals a widely dilated esophagus with a smoothly tapering distal esophagus. There appears to be partially digested food present in the esophagus. Which of the following would most likely be seen on esophageal manometry?
Correct
The correct answer is A. This question describes a typical presentation of achalasia. Achalasia is a neurogenic esophageal disorder thought to be caused by a malfunction of the myenteric plexus of the esophagus. The result is denervation of the distal esophageal muscle with resulting impaired esophageal peristalsis. The characteristic findings on esophageal manometry are diminished or absent peristalsis in the body of the esophagus and a high resting lower esophageal pressure. The latter finding is related to a failure of the usual relaxation of the sphincter as the bolus of food reaches it.
Choices B and E are wrong, because the upper esophageal sphincter is usually unaffected in this condition.
Choice C is wrong because peristalsis is decreased or absent in the mid-esophagus in achalasia.
Choice D is wrong because resting lower esophageal pressure is characteristically high in achalasia.
Incorrect
The correct answer is A. This question describes a typical presentation of achalasia. Achalasia is a neurogenic esophageal disorder thought to be caused by a malfunction of the myenteric plexus of the esophagus. The result is denervation of the distal esophageal muscle with resulting impaired esophageal peristalsis. The characteristic findings on esophageal manometry are diminished or absent peristalsis in the body of the esophagus and a high resting lower esophageal pressure. The latter finding is related to a failure of the usual relaxation of the sphincter as the bolus of food reaches it.
Choices B and E are wrong, because the upper esophageal sphincter is usually unaffected in this condition.
Choice C is wrong because peristalsis is decreased or absent in the mid-esophagus in achalasia.
Choice D is wrong because resting lower esophageal pressure is characteristically high in achalasia.
-
Question 11 of 151
11. Question
45-year-old man undergoes a routine examination. While the physical examination is unrevealing, a hematocrit performed in the physician’s office gives a value of 25%. Review of the peripheral smear reveals smaller-than-normal erythrocytes. The cells vary in size, and some have abnormal shapes. The cells do not appear paler than normal. Reticulocytes are decreased. Assuming that this patient has only a single cause for his anemia, which of the following is most likely to be seen on further evaluation?
Correct
The correct choice is A. This patient has a microcytic,normochromic anemia and is not obviously ill on physical examination. The overwhelmingly most likely diagnosis is iron-deficiency anemia, which is, in turn, almost always due to bleeding in adults on a typical American diet. Occult GI bleeding is a common source in both men and women. Menstrual disorders are also important causes of iron deficiency in women.
Low iron binding capacity (choice B) is seen in the anemia of chronic disease.
Low folate (choice C) produces a megaloblastic anemia.
Low mean corpuscular hemoglobin concentration (MCHC; choiceD) is seen in hypochromic anemias.
Low vitamin B12 (choice E) produces a megaloblastic anemia.
Incorrect
The correct choice is A. This patient has a microcytic,normochromic anemia and is not obviously ill on physical examination. The overwhelmingly most likely diagnosis is iron-deficiency anemia, which is, in turn, almost always due to bleeding in adults on a typical American diet. Occult GI bleeding is a common source in both men and women. Menstrual disorders are also important causes of iron deficiency in women.
Low iron binding capacity (choice B) is seen in the anemia of chronic disease.
Low folate (choice C) produces a megaloblastic anemia.
Low mean corpuscular hemoglobin concentration (MCHC; choiceD) is seen in hypochromic anemias.
Low vitamin B12 (choice E) produces a megaloblastic anemia.
-
Question 12 of 151
12. Question
A 23-year old-dancer presents with a chief complaint of weakness. She denies any other symptoms, including nausea or vomiting. She denies diarrhea. Her blood pressure is 80/40 mm Hg. There is no edema and the lungs are clear. Laboratory analysis of serum shows:
Sodium 126 mEq/L
Potassium 2.2 mEq/L
Bicarbonate 29 mEq/L
Magnesium 2.0 mg/dL
Calcium 9.0 mg/dLThe most likely cause of the patient’s weakness is an abnormality in which of the following?
Correct
The correct answer is D. The low potassium is the most likely cause of the weakness. The patient may be a diuretic abuser or may vomit as part of an eating disorder.
The high bicarbonate (choice A) may reflect an alkalosis, but should not cause symptoms like this.
The magnesium (choice C) and calcium (choice B) values are close to the normal range.
The low sodium (choice E) is not likely to cause weakness.
Incorrect
The correct answer is D. The low potassium is the most likely cause of the weakness. The patient may be a diuretic abuser or may vomit as part of an eating disorder.
The high bicarbonate (choice A) may reflect an alkalosis, but should not cause symptoms like this.
The magnesium (choice C) and calcium (choice B) values are close to the normal range.
The low sodium (choice E) is not likely to cause weakness.
-
Question 13 of 151
13. Question
One week after an upper respiratory infection, an adult develops conjunctival hyperemia, watery discharge, and ocular irritation. While both eyes are involved by the time a physician was consulted, the symptoms began in one eye. On the morning of the doctor’s visit, the patient had difficulty opening his eyelids on awakening as they were “glued shut”. Physical examination demonstrates hyperemic bulbar and tarsal conjunctivae. No purulentfluid is seen. The preauricular lymph node on one side is enlarged. Which of the following pathogens would be most likely to cause these symptoms?
Correct
The correct answer is A. This is viral conjunctivitis, which is an acute conjunctival inflammation most often caused by adenovirus. Most of the patients with this condition have either been recently exposed to someone with viral conjunctivitis or have recently had a viral upper respiratory infection. The presentation illustrated in the question stem is typical. Severe cases also additionally have photophobia and a foreign body sensation. Pseudomembranes of fibrin and inflammatory cells on the conjunctival surfaces can occur. Corneal involvement can leave residual scarring visible by slit lamp for up to 2 years after a severe case. Cases are self-limited but tend to be extremely contagious, so care should be taken to have both the patient and the physician wash their hands very thoroughly after touching the face.
Herpes virus (choices B and C) can cause corneal ulceration, hyperkeratosis, or scarring.
Neisseria gonorrhoeae(choice D) can cause conjunctivitis in both adults and neonates, and produces a purulent, rather than a watery discharge.
Staphylococcus aureus(choice E) is a common cause of bacterial conjunctivitis, and produces a purulent, rather than a watery discharge.
Incorrect
The correct answer is A. This is viral conjunctivitis, which is an acute conjunctival inflammation most often caused by adenovirus. Most of the patients with this condition have either been recently exposed to someone with viral conjunctivitis or have recently had a viral upper respiratory infection. The presentation illustrated in the question stem is typical. Severe cases also additionally have photophobia and a foreign body sensation. Pseudomembranes of fibrin and inflammatory cells on the conjunctival surfaces can occur. Corneal involvement can leave residual scarring visible by slit lamp for up to 2 years after a severe case. Cases are self-limited but tend to be extremely contagious, so care should be taken to have both the patient and the physician wash their hands very thoroughly after touching the face.
Herpes virus (choices B and C) can cause corneal ulceration, hyperkeratosis, or scarring.
Neisseria gonorrhoeae(choice D) can cause conjunctivitis in both adults and neonates, and produces a purulent, rather than a watery discharge.
Staphylococcus aureus(choice E) is a common cause of bacterial conjunctivitis, and produces a purulent, rather than a watery discharge.
-
Question 14 of 151
14. Question
A 6-week-old girl comes to medical attention because of a bright red plaque on her neck. The plaque is slightly raised, measures 2 cm in greatest diameter, and blanches partially on pressure. The mother reports that the lesion appeared in the second or third week and has been gradually increasing in size. Which of the following is the most appropriate next step in management?
Correct
The correct answer is A. The gross appearance of this red plaque, combined with its development and further growth beginning in the first month of life, makes a diagnosis of hemangioma most probable. Hemangiomas may involve the skin or deep organs, and may be small or rather extensive lesions. Skin hemangiomas become apparent in the first month of life, being occasionally preceded by an erythematous macule. They grow for 6-10 months, reach a stable size, and then undergo spontaneous regression. The great majority of skin hemangiomas disappear entirely by age 9. Some hemangiomas need further diagnostic evaluation or treatment either because of their location, or because of the likelihood of being associated with other congenital anomalies.
MRI and/or CT studies (choice B), for example, would be necessary in cases of extensive cervical hemangiomas, periorbital hemangiomas, or hemangiomas located in the lumbosacral region. These hemangiomas may be associated with underlying abnormalities, such as spinal dysraphism, posterior fossa malformations, or extension into orbital or laryngeal structures.
Referral to a dermatologist for biopsy (choice C) is rarely necessary when the gross features are characteristic of classic hemangiomas.
Treatment with corticosteroids or interferon-alfa (choice D) may be indicated in cases of periorbital hemangiomas. These lesions may extend into the orbit and lead to progressive visual impairment. Ophthalmologic evaluation and follow-up is warranted.
Surgical excision (choice E) is never recommended. Further growth of hemangiomas can be prevented or slowed by corticosteroids or interferon-alfa. Laser treatment is useful for ulcerated hemangiomas.
Incorrect
The correct answer is A. The gross appearance of this red plaque, combined with its development and further growth beginning in the first month of life, makes a diagnosis of hemangioma most probable. Hemangiomas may involve the skin or deep organs, and may be small or rather extensive lesions. Skin hemangiomas become apparent in the first month of life, being occasionally preceded by an erythematous macule. They grow for 6-10 months, reach a stable size, and then undergo spontaneous regression. The great majority of skin hemangiomas disappear entirely by age 9. Some hemangiomas need further diagnostic evaluation or treatment either because of their location, or because of the likelihood of being associated with other congenital anomalies.
MRI and/or CT studies (choice B), for example, would be necessary in cases of extensive cervical hemangiomas, periorbital hemangiomas, or hemangiomas located in the lumbosacral region. These hemangiomas may be associated with underlying abnormalities, such as spinal dysraphism, posterior fossa malformations, or extension into orbital or laryngeal structures.
Referral to a dermatologist for biopsy (choice C) is rarely necessary when the gross features are characteristic of classic hemangiomas.
Treatment with corticosteroids or interferon-alfa (choice D) may be indicated in cases of periorbital hemangiomas. These lesions may extend into the orbit and lead to progressive visual impairment. Ophthalmologic evaluation and follow-up is warranted.
Surgical excision (choice E) is never recommended. Further growth of hemangiomas can be prevented or slowed by corticosteroids or interferon-alfa. Laser treatment is useful for ulcerated hemangiomas.
-
Question 15 of 151
15. Question
51-year-old man presents to his physician for evaluation of his risk for heart disease. The patient is very concerned about his chance for a heart attack because his father died at age 53 from a massive infarction. The patient reports that he eats well, exercises regularly, and has been told by his workplace screening program that he has “normal” cholesterol levels. On questioning, he reports that he had previously smoked one pack per day for about 20 years but quit 2 years ago. How many years must this patient have stopped smoking before his tobacco use no longer counts as a risk factor?
Correct
The correct answer is D. The major cardiac risk factors are family history, age, tobacco use, hypertension, diabetes, and low HDL. Tobacco use counts, even if it is no longer current, for 15 years.
Incorrect
The correct answer is D. The major cardiac risk factors are family history, age, tobacco use, hypertension, diabetes, and low HDL. Tobacco use counts, even if it is no longer current, for 15 years.
-
Question 16 of 151
16. Question
13-year-old boy is seen for right knee pain that has persisted for the past 2 months despite the use of over-the-counter analgesics. His mother states that he has been limping since he started to have the pain. The pain is of insidious onset, but its intensity sometimes suddenly increases. On physical examination, the boy is obese; his weight is higher than the 90th percentile. Flexion of the right hip causes marked outward rotation and reproduces the pain. Internal rotation of the right hip is limited, and the right leg is slightly shortened compared with the left leg. Which of the following is the most likely diagnosis?
Correct
The correct answer is D. Slipped capital femoral epiphysis (SCFE) is a relatively common hip problem, usually seen in the adolescent near the time of peak growth velocity. It is characterized by displacement of the femoral epiphysis from the femoral neck through the growth plate. The displacement can be either sudden or gradual. A slipped epiphysis can later develop in the opposite hip in 36% of cases. SCFE occurs more commonly in younger children, and boys are affected more than girls. Mechanical stress during the growth spurt is an important cause of SCFE, and at least one half the children with the condition are obese. SCFE occurs with an increased frequency in association with endocrine disorders, especially hypothyroidism, panhypopituitarism, and renal disease. Because of its frequency, the risk for degenerative arthritis, and the possibility of sudden instability and vascular damage, early detection of SCFE is extremely important.
Clinical findings can vary depending on whether the slip is stable or unstable. In a stable slip, an obese adolescent boy may complain of activity-related hip pain that is relieved by rest. There may be a mild limp after strenuous activity. These symptoms may last weeks or months. Physical examination shows pain with hip motion, restriction of flexion, and, particularly, internal rotation while the hip is flexed to 90 degrees. The diagnosis can be elusive, since referral of the pain may be to the distal thigh, presenting as knee pain. All children who complain of knee pain should have their hip examined. Some children with an extremely externally rotated gait have foot pain as their presenting complaint. The unstable slip does not allow the child to bear weight, even with crutches. This should be considered the equivalent of an acute, displaced fracture of the hip and should be treated urgently. The child is in extreme pain and usually gives a history of a fall from a bicycle or a tree; however, the fall may also be trivial if the epiphysis had preexisting weakness. Radiographs confirm the diagnosis, and the patient must be urgently admitted to the hospital for treatment. The risk for avascular necrosis, even with immediate treatment, is as high as 47%.
Radiographs are obtained with the patient in the supine position for anteroposterior (AP) and frog lateral pelvis films. Once the diagnosis of a stable slipped epiphysis is made in a walking child, surgery should be performed without delay. The most common procedure is percutaneous pin or screw fixation between the femoral neck and the center of the epiphysis.
Juvenile rheumatoid arthritis (choice A) is the most prevalent chronic pediatric rheumatologic disease. The arthritis is frequently symmetric and predominantly affects larger joints. Patients can have a fever and an elevated erythrocyte sedimentation rate and white blood cell count.
Legg-Calve-Perthes disease (choice B) is avascular necrosis of the femoral epiphysis. It occurs more frequently in males and young children aged 4-8 years.
It usually presents with limping and knee pain. Septic arthritis (choice C) presents with a painful and swollen joint, as well as fever. It is diagnosed by aspiration of the synovial fluid, which will yield a very high white cell count and a pathogen on culture.
Toxic synovitis (choice E) is a benign and common condition that often presents with hip pain, knee pain, and limping. It is usually preceded by a viral illness.
Incorrect
The correct answer is D. Slipped capital femoral epiphysis (SCFE) is a relatively common hip problem, usually seen in the adolescent near the time of peak growth velocity. It is characterized by displacement of the femoral epiphysis from the femoral neck through the growth plate. The displacement can be either sudden or gradual. A slipped epiphysis can later develop in the opposite hip in 36% of cases. SCFE occurs more commonly in younger children, and boys are affected more than girls. Mechanical stress during the growth spurt is an important cause of SCFE, and at least one half the children with the condition are obese. SCFE occurs with an increased frequency in association with endocrine disorders, especially hypothyroidism, panhypopituitarism, and renal disease. Because of its frequency, the risk for degenerative arthritis, and the possibility of sudden instability and vascular damage, early detection of SCFE is extremely important.
Clinical findings can vary depending on whether the slip is stable or unstable. In a stable slip, an obese adolescent boy may complain of activity-related hip pain that is relieved by rest. There may be a mild limp after strenuous activity. These symptoms may last weeks or months. Physical examination shows pain with hip motion, restriction of flexion, and, particularly, internal rotation while the hip is flexed to 90 degrees. The diagnosis can be elusive, since referral of the pain may be to the distal thigh, presenting as knee pain. All children who complain of knee pain should have their hip examined. Some children with an extremely externally rotated gait have foot pain as their presenting complaint. The unstable slip does not allow the child to bear weight, even with crutches. This should be considered the equivalent of an acute, displaced fracture of the hip and should be treated urgently. The child is in extreme pain and usually gives a history of a fall from a bicycle or a tree; however, the fall may also be trivial if the epiphysis had preexisting weakness. Radiographs confirm the diagnosis, and the patient must be urgently admitted to the hospital for treatment. The risk for avascular necrosis, even with immediate treatment, is as high as 47%.
Radiographs are obtained with the patient in the supine position for anteroposterior (AP) and frog lateral pelvis films. Once the diagnosis of a stable slipped epiphysis is made in a walking child, surgery should be performed without delay. The most common procedure is percutaneous pin or screw fixation between the femoral neck and the center of the epiphysis.
Juvenile rheumatoid arthritis (choice A) is the most prevalent chronic pediatric rheumatologic disease. The arthritis is frequently symmetric and predominantly affects larger joints. Patients can have a fever and an elevated erythrocyte sedimentation rate and white blood cell count.
Legg-Calve-Perthes disease (choice B) is avascular necrosis of the femoral epiphysis. It occurs more frequently in males and young children aged 4-8 years.
It usually presents with limping and knee pain. Septic arthritis (choice C) presents with a painful and swollen joint, as well as fever. It is diagnosed by aspiration of the synovial fluid, which will yield a very high white cell count and a pathogen on culture.
Toxic synovitis (choice E) is a benign and common condition that often presents with hip pain, knee pain, and limping. It is usually preceded by a viral illness.
-
Question 17 of 151
17. Question
A 54-year-old woman with a long history of emphysema presents because of an exacerbation of her cough and dyspnea. On physical examination, her blood pressure is 126/64 mm Hg, pulse is 82/min, and respirations are 24/min. On lung examination, there are loud expiratory wheezes and rhonchi. The cardiac examination is normal. An arterial blood gas is performed. Which of the following results would most likely be expected?
Correct
The correct answer is C. This patient has an acute exacerbation of her chronic obstructive pulmonary disease (COPD). On the basis of the history and physical examination, she would be expected to have a compensated respiratory acidosis because CO2 retention. Furthermore, she would be expected to have evidence of mild hypoxia. With respirations of 24/min, which is high but not extremely high, she would not be expected to show the severe levels of acidosis and CO2 retention illustrated in choice A.
The patient is sufficiently symptomatic that the near normal pO2 of 94 seen in choice B would be unlikely.
Although asthmatics may present during an acute exacerbation with a respiratory alkalosis (choices D and E), in a patient with underlying COPD, there is usually a baseline respiratory acidosis.Incorrect
The correct answer is C. This patient has an acute exacerbation of her chronic obstructive pulmonary disease (COPD). On the basis of the history and physical examination, she would be expected to have a compensated respiratory acidosis because CO2 retention. Furthermore, she would be expected to have evidence of mild hypoxia. With respirations of 24/min, which is high but not extremely high, she would not be expected to show the severe levels of acidosis and CO2 retention illustrated in choice A.
The patient is sufficiently symptomatic that the near normal pO2 of 94 seen in choice B would be unlikely.
Although asthmatics may present during an acute exacerbation with a respiratory alkalosis (choices D and E), in a patient with underlying COPD, there is usually a baseline respiratory acidosis. -
Question 18 of 151
18. Question
A 45-year-old woman presents to clinic with a 4-month history of headaches and changes in her vision. She has been previously healthy and has not been on any medications. On examination, she is found to have a small field defect in both eyes. The diagnosis of a pituitary microadenoma is entertained. Which of the following tests would be the most sensitive in diagnosis of such an entity?
Correct
The correct answer is C. MRI with gadolinium is considered the most sensitive test for detecting microadenoma. The test can reveal microadenomas in 20% of normal women.
CT scans can be quite sensitive in the detection of microadenomas, but MRIs are even more so (choice A).
Insulin-resistance as a result of impaired growth hormone response to insulin-induced hypoglycemia is not as common (choice B).
The serum prolactin level is elevated due to hypersecretion in 30-50% of patients (choice D).
For the optic chiasm to be compressed in order to cause visual field changes, the microadenoma would have to be quite large, and a screening test based on this finding would be ineffective (choice E).Incorrect
The correct answer is C. MRI with gadolinium is considered the most sensitive test for detecting microadenoma. The test can reveal microadenomas in 20% of normal women.
CT scans can be quite sensitive in the detection of microadenomas, but MRIs are even more so (choice A).
Insulin-resistance as a result of impaired growth hormone response to insulin-induced hypoglycemia is not as common (choice B).
The serum prolactin level is elevated due to hypersecretion in 30-50% of patients (choice D).
For the optic chiasm to be compressed in order to cause visual field changes, the microadenoma would have to be quite large, and a screening test based on this finding would be ineffective (choice E). -
Question 19 of 151
19. Question
14-year-old boy is brought to the emergency room because of persistent midepigastric pain for two days. The pain is getting no better, yet it is no worse, and radiates to his back. The boy also has had fever, as well as nausea and vomiting that is worse when his temperature rises in the afternoon. On examination, his temperature is 39.2 C (102.5 F) and there is marked upper abdominal tenderness with guarding. Mild abdominal distention is present with no audible bowel sounds. A complete blood count reveals an elevated leukocyte count and a normal serum amylase. Which of the following is the most likely diagnosis?
Correct
The correct answer is D. Pancreatitis is not always an easy diagnosis. The history of this boy is particularly suspect, even with a normal serum amylase. Nearly one-third of all patients with acute pancreatitis have a normal serum amylase. An abdominal ultrasound may be useful in revealing an enlarged pancreas.
This young man is unlikely to have Fitz-Hugh-Curtis syndrome (choice A). Classically, Fitz-Hugh-Curtis syndrome is an extrapelvic manifestation of pelvic inflammatory disease in sexually active women. It is a perihepatitis consisting of adhesions between the liver capsule and the diaphragm or the anterior peritoneal surface. It was originally thought to be caused solely by Neisseria gonorrhoeae, but recent studies have shown that Chlamydia trachomatis and other organisms may also be etiologies. The Fitz-Hugh-Curtis syndrome has rarely been reported in men, so a sexual history should be obtained in this case.
Intussusception (choice B) is not high on the list of differential diagnoses. The typical age for intussusception is in infancy, especially 6 to 12 months of age. This young man has persistent abdominal pain with fever, making this an unlikely diagnosis.
Wilms tumor (choice C) is also unlikely in this case scenario. Wilms tumor usually occurs in early childhood, age 2 or 3 years. Although Wilms tumor is more prevalent in males, the usual presenting signs include an asymptomatic abdominal mass and hematuria.
Pyelonephritis (choice E) typically presents with high fever, chills, and back pain. Although a urinalysis should be performed in this young man, the symptoms of midepigastric pain suggest an upper gastrointestinal problem, not pyelonephritis.Incorrect
The correct answer is D. Pancreatitis is not always an easy diagnosis. The history of this boy is particularly suspect, even with a normal serum amylase. Nearly one-third of all patients with acute pancreatitis have a normal serum amylase. An abdominal ultrasound may be useful in revealing an enlarged pancreas.
This young man is unlikely to have Fitz-Hugh-Curtis syndrome (choice A). Classically, Fitz-Hugh-Curtis syndrome is an extrapelvic manifestation of pelvic inflammatory disease in sexually active women. It is a perihepatitis consisting of adhesions between the liver capsule and the diaphragm or the anterior peritoneal surface. It was originally thought to be caused solely by Neisseria gonorrhoeae, but recent studies have shown that Chlamydia trachomatis and other organisms may also be etiologies. The Fitz-Hugh-Curtis syndrome has rarely been reported in men, so a sexual history should be obtained in this case.
Intussusception (choice B) is not high on the list of differential diagnoses. The typical age for intussusception is in infancy, especially 6 to 12 months of age. This young man has persistent abdominal pain with fever, making this an unlikely diagnosis.
Wilms tumor (choice C) is also unlikely in this case scenario. Wilms tumor usually occurs in early childhood, age 2 or 3 years. Although Wilms tumor is more prevalent in males, the usual presenting signs include an asymptomatic abdominal mass and hematuria.
Pyelonephritis (choice E) typically presents with high fever, chills, and back pain. Although a urinalysis should be performed in this young man, the symptoms of midepigastric pain suggest an upper gastrointestinal problem, not pyelonephritis. -
Question 20 of 151
20. Question
A newborn girl is noted to be drooling saliva, and she chokes violently when she is first fed. On physical examination, she is found to have abdominal distention and an imperforate anus. There is no fistula to the perineum or vagina. Examination of the urine reveals no meconium in it. Echocardiogram and renal sonogram are reported as negative for other congenital defects. X-ray films show abundant gas in the gastrointestinal tract. Pictures taken with a metal marker taped to the anus, and the baby hanging upside down, show that there is a significant distance (2.5 cm) between the blind end of the rectum and the anal marker. Before a diverting colostomy is performed, which of the following steps should be taken?
Correct
The correct answer is A. Although the physical finding of the imperforate anus has given you a diagnosis, the presentation with drooling and choking is classic for esophageal atresia, which is often an associated anomaly (part of the “VACTER” constellation). In fact, the abundance of gas in the gastrointestinal tract is also typical, as the babies get it by way of the distal tracheoesophageal fistula that accompanies the majority of esophageal atresias. The nasogastric tube coiled on itself would confirm the suspected diagnosis, which would significantly alter the surgical plans.
Barium injected through the anal dimple (choice B) would probably end up in the tissues, where it would be very irritating. In the absence of a fistula, imperforate anus is not likely to be low. The study would not help define the imperforate anus and would do nothing to diagnose the esophageal problem.
Barium swallow (choice C) would demonstrate the esophageal atresia, but at the cost of ending up in the tracheobronchial tree as the baby vomits and aspirates it. Barium is very irritating in that location. If contrast material is needed to demonstrate the atresia, water soluble contrast material would be preferred, and it would have to be promptly sucked out before it also ended up in the lungs.
Decompressing the gastrointestinal tract (choice D) would not be possible by nasogastric suction if the esophagus indeed is atretic. If it is patent, the colostomy can be done without prior decompression. If there is a tracheoesophageal fistula, and for some reason it cannot be promptly repaired, a gastrostomy would be required to protect the lungs from aspiration of gastric acid.
Attempts to reconnect the anal canal (choice E), when all evidence points to a high blind end, would be pointless and would do nothing to diagnose the other potentially lethal anomaly affecting the esophagus and tracheobronchial treeIncorrect
The correct answer is A. Although the physical finding of the imperforate anus has given you a diagnosis, the presentation with drooling and choking is classic for esophageal atresia, which is often an associated anomaly (part of the “VACTER” constellation). In fact, the abundance of gas in the gastrointestinal tract is also typical, as the babies get it by way of the distal tracheoesophageal fistula that accompanies the majority of esophageal atresias. The nasogastric tube coiled on itself would confirm the suspected diagnosis, which would significantly alter the surgical plans.
Barium injected through the anal dimple (choice B) would probably end up in the tissues, where it would be very irritating. In the absence of a fistula, imperforate anus is not likely to be low. The study would not help define the imperforate anus and would do nothing to diagnose the esophageal problem.
Barium swallow (choice C) would demonstrate the esophageal atresia, but at the cost of ending up in the tracheobronchial tree as the baby vomits and aspirates it. Barium is very irritating in that location. If contrast material is needed to demonstrate the atresia, water soluble contrast material would be preferred, and it would have to be promptly sucked out before it also ended up in the lungs.
Decompressing the gastrointestinal tract (choice D) would not be possible by nasogastric suction if the esophagus indeed is atretic. If it is patent, the colostomy can be done without prior decompression. If there is a tracheoesophageal fistula, and for some reason it cannot be promptly repaired, a gastrostomy would be required to protect the lungs from aspiration of gastric acid.
Attempts to reconnect the anal canal (choice E), when all evidence points to a high blind end, would be pointless and would do nothing to diagnose the other potentially lethal anomaly affecting the esophagus and tracheobronchial tree -
Question 21 of 151
21. Question
A 22-year-old man comes to the emergency department because of dyspnea, palpitations, and a headache. These symptoms came on soon after he took trimethoprim-sulfamethoxazole for a urinary tract infection. Laboratory studies show a normochromic, normocytic anemia. A peripheral blood smear reveals Heinz bodies. Which of the following is the most likely cause of this patient’s anemia?
Correct
The correct answer is C. This patient has glucose-6-phosphate dehydrogenase (G6PD) deficiency, which is an X-linked disorder that leads to hemolytic crises within hours of exposure to oxidant stress. The most common stressors are viral and bacterial infections, sulfa drugs, quinines, and fava beans. During an acute hemolytic crisis, hemoglobin becomes denatured and leads to the formation of Heinz bodies. The diagnosis is made by the demonstration of Heinz bodies during an acute crisis, and low levels of G6PD during normal times. The treatment includes maintaining adequate urine output and the prevention of future episodes.
Lead poisoning (choice A) leads to a normochromic, normocytic anemia with basophilic stippling. The clinical features include abdominal pain, headache, irritability, and peripheral motor neuropathy. Treatment includes the use of chelating agents.
Folate deficiency (choice B) leads to megaloblastic anemia and is most common in alcoholics.
Hereditary spherocytosis (choice D) is an inherited membrane disorder that leads to hemolytic anemia and red blood cell swelling. Small, round, hyperchromatic red cells without a central area of pallor are seen on blood smears.
Occult blood loss (choice E) leads to chronic iron loss and microcytic anemia. The symptoms include a gradual onset of weakness and fatigue. Management includes the diagnosis and control of the underlying disorder.Incorrect
The correct answer is C. This patient has glucose-6-phosphate dehydrogenase (G6PD) deficiency, which is an X-linked disorder that leads to hemolytic crises within hours of exposure to oxidant stress. The most common stressors are viral and bacterial infections, sulfa drugs, quinines, and fava beans. During an acute hemolytic crisis, hemoglobin becomes denatured and leads to the formation of Heinz bodies. The diagnosis is made by the demonstration of Heinz bodies during an acute crisis, and low levels of G6PD during normal times. The treatment includes maintaining adequate urine output and the prevention of future episodes.
Lead poisoning (choice A) leads to a normochromic, normocytic anemia with basophilic stippling. The clinical features include abdominal pain, headache, irritability, and peripheral motor neuropathy. Treatment includes the use of chelating agents.
Folate deficiency (choice B) leads to megaloblastic anemia and is most common in alcoholics.
Hereditary spherocytosis (choice D) is an inherited membrane disorder that leads to hemolytic anemia and red blood cell swelling. Small, round, hyperchromatic red cells without a central area of pallor are seen on blood smears.
Occult blood loss (choice E) leads to chronic iron loss and microcytic anemia. The symptoms include a gradual onset of weakness and fatigue. Management includes the diagnosis and control of the underlying disorder. -
Question 22 of 151
22. Question
A 22-year-old man comes to the emergency department because of dyspnea, palpitations, and a headache. These symptoms came on soon after he took trimethoprim-sulfamethoxazole for a urinary tract infection. Laboratory studies show a normochromic, normocytic anemia. A peripheral blood smear reveals Heinz bodies. Which of the following is the most likely cause of this patient’s anemia?
Correct
The correct answer is C. This patient has glucose-6-phosphate dehydrogenase (G6PD) deficiency, which is an X-linked disorder that leads to hemolytic crises within hours of exposure to oxidant stress. The most common stressors are viral and bacterial infections, sulfa drugs, quinines, and fava beans. During an acute hemolytic crisis, hemoglobin becomes denatured and leads to the formation of Heinz bodies. The diagnosis is made by the demonstration of Heinz bodies during an acute crisis, and low levels of G6PD during normal times. The treatment includes maintaining adequate urine output and the prevention of future episodes.
Lead poisoning (choice A) leads to a normochromic, normocytic anemia with basophilic stippling. The clinical features include abdominal pain, headache, irritability, and peripheral motor neuropathy. Treatment includes the use of chelating agents.
Folate deficiency (choice B) leads to megaloblastic anemia and is most common in alcoholics.
Hereditary spherocytosis (choice D) is an inherited membrane disorder that leads to hemolytic anemia and red blood cell swelling. Small, round, hyperchromatic red cells without a central area of pallor are seen on blood smears.
Occult blood loss (choice E) leads to chronic iron loss and microcytic anemia. The symptoms include a gradual onset of weakness and fatigue. Management includes the diagnosis and control of the underlying disorder.Incorrect
The correct answer is C. This patient has glucose-6-phosphate dehydrogenase (G6PD) deficiency, which is an X-linked disorder that leads to hemolytic crises within hours of exposure to oxidant stress. The most common stressors are viral and bacterial infections, sulfa drugs, quinines, and fava beans. During an acute hemolytic crisis, hemoglobin becomes denatured and leads to the formation of Heinz bodies. The diagnosis is made by the demonstration of Heinz bodies during an acute crisis, and low levels of G6PD during normal times. The treatment includes maintaining adequate urine output and the prevention of future episodes.
Lead poisoning (choice A) leads to a normochromic, normocytic anemia with basophilic stippling. The clinical features include abdominal pain, headache, irritability, and peripheral motor neuropathy. Treatment includes the use of chelating agents.
Folate deficiency (choice B) leads to megaloblastic anemia and is most common in alcoholics.
Hereditary spherocytosis (choice D) is an inherited membrane disorder that leads to hemolytic anemia and red blood cell swelling. Small, round, hyperchromatic red cells without a central area of pallor are seen on blood smears.
Occult blood loss (choice E) leads to chronic iron loss and microcytic anemia. The symptoms include a gradual onset of weakness and fatigue. Management includes the diagnosis and control of the underlying disorder. -
Question 23 of 151
23. Question
A 28-year-old patient with end-stage renal disease (ESRD) on continuous ambulatory peritoneal dialysis (CAPD) for two months presents with fever, abdominal pain and cloudy dialysis fluid. There is no diarrhea or vomiting and the pain has been present for about 12 hours. The patient has ESRD secondary to chronic glomerulonephritis, there is no history of diabetes, urinary infections or antibiotic use. Examination reveals a temperature of 38.9 C (102 F), and blood pressure of 110/70 mm Hg. The throat is clear, as are the lungs. Cardiac examination reveals a grade 2/6 systolic murmur. Abdominal examination reveals decreased bowel sounds with diffuse tenderness. There is mild rebound. There is no edema or skin rash. A complete blood count shows a leukocyte count of 14,200/mm3, hemoglobin is 12.5 g/dL. Peritoneal fluid is cloudy with 1,000 white blood cells, 85% of which are polymorphonuclear leukocytes. Gram’s stain of fluid is negative. Cultures of blood and peritoneal dialysis fluid are taken. Which of the following is the most appropriate initial step in management?
Correct
The correct answer is C. Peritonitis in a patient on CAPD is usually due to gram-positive pathogens such as Staphylococcus aureus or epidermidis. It is usually characterized by abdominal pain and over 100 white blood cells (typically polymorphonuclear leukocytes) in a sample of peritoneal dialysis fluid. Intravenous vancomycin would be a reasonable treatment to cover gram-positive pathogens.
Fluconazole (choice A) would be indicated for a fungal infection. Fungal peritonitis is not usually seen until patients have been treated with multiple antibiotics or are further immunosuppressed. Immediate removal of the dialysis catheter (choice B) is usually not needed unless the patient has a peritonitis that has not improved with a trial of antibiotics.
Intravenous gentamicin (choice D) has good gram-negative coverage but would not be an ideal drug to cover Staphylococcus.
Ciprofloxacin (choice E) would be a very broad spectrum antibiotic that would not be a first choice as a single antibiotic to treat staphylococcal peritonitis. Further, the oral route may not be adequate as patients with peritonitis may have nausea and vomitingIncorrect
The correct answer is C. Peritonitis in a patient on CAPD is usually due to gram-positive pathogens such as Staphylococcus aureus or epidermidis. It is usually characterized by abdominal pain and over 100 white blood cells (typically polymorphonuclear leukocytes) in a sample of peritoneal dialysis fluid. Intravenous vancomycin would be a reasonable treatment to cover gram-positive pathogens.
Fluconazole (choice A) would be indicated for a fungal infection. Fungal peritonitis is not usually seen until patients have been treated with multiple antibiotics or are further immunosuppressed. Immediate removal of the dialysis catheter (choice B) is usually not needed unless the patient has a peritonitis that has not improved with a trial of antibiotics.
Intravenous gentamicin (choice D) has good gram-negative coverage but would not be an ideal drug to cover Staphylococcus.
Ciprofloxacin (choice E) would be a very broad spectrum antibiotic that would not be a first choice as a single antibiotic to treat staphylococcal peritonitis. Further, the oral route may not be adequate as patients with peritonitis may have nausea and vomiting -
Question 24 of 151
24. Question
A 28-month-old female has been living with her mother in a shelter for homeless women and children. She is brought to the Emergency Department in status epilepticus, which is stopped with intravenous lorazepam. She is placed on a cardiac monitor and a wide complex tachycardia is noted. The ventricular tachycardia reverts to sinus tachycardia after defibrillation is performed. Her temperature is 38.5 C (101.3 F), blood pressure is 120/80 mm Hg, pulse is 195, and respirations are 26. Physical examination reveals a lethargic, pale toddler with dilated and reactive pupils, dry mucous membranes, shallow respirations, diaphoresis and brisk deep tendon reflexes. A urine toxicology screen is most likely to detect which of the following substances?
Correct
The correct answer is B. This patient’s clinical picture is consistent with acute cocaine ingestion. Cocaine is a CNS stimulant that causes increased heart rate, hypertension and hyperactive reflexes. Other substances of abuse can cause cardiac arrhythmias and seizures but of the answer choices, only cocaine causes mydriasis or dilated pupils. Treatment is supportive and may include anti-arrhythmics, anti-hypertensives, cooling blankets and sedatives.
Barbiturates (choice A) are CNS depressants and would cause hypotension, normal or constricted pupils and diminished reflexes. Other classic features of barbiturate overdose include ataxia, slurred speech and nystagmus.
Heroin (choice C) is an opiate and would produce symptoms of CNS depression (hypotension, diminished or absent reflexes, hypoventilation, hypothermia) and constricted pupils. Other symptoms of opiate intoxication include nausea, vomiting, euphoria, seizures and coma.
Marijuana (choice D) ingestion or inhalation causes relatively benign symptoms that include euphoria, hunger, tachycardia and injected conjunctivae. Pupils usually remain normal.
PCP (choice E) is a hallucinogen that causes CNS stimulation (hypertension, tachycardia, brisk reflexes, hyperthermia). Other hallucinogens such as LSD and mescaline cause dilated pupils but PCP has an effect of constricting pupils. Seizures and coma are more common with PCP and intracranial hemorrhages have been reported.Incorrect
The correct answer is B. This patient’s clinical picture is consistent with acute cocaine ingestion. Cocaine is a CNS stimulant that causes increased heart rate, hypertension and hyperactive reflexes. Other substances of abuse can cause cardiac arrhythmias and seizures but of the answer choices, only cocaine causes mydriasis or dilated pupils. Treatment is supportive and may include anti-arrhythmics, anti-hypertensives, cooling blankets and sedatives.
Barbiturates (choice A) are CNS depressants and would cause hypotension, normal or constricted pupils and diminished reflexes. Other classic features of barbiturate overdose include ataxia, slurred speech and nystagmus.
Heroin (choice C) is an opiate and would produce symptoms of CNS depression (hypotension, diminished or absent reflexes, hypoventilation, hypothermia) and constricted pupils. Other symptoms of opiate intoxication include nausea, vomiting, euphoria, seizures and coma.
Marijuana (choice D) ingestion or inhalation causes relatively benign symptoms that include euphoria, hunger, tachycardia and injected conjunctivae. Pupils usually remain normal.
PCP (choice E) is a hallucinogen that causes CNS stimulation (hypertension, tachycardia, brisk reflexes, hyperthermia). Other hallucinogens such as LSD and mescaline cause dilated pupils but PCP has an effect of constricting pupils. Seizures and coma are more common with PCP and intracranial hemorrhages have been reported. -
Question 25 of 151
25. Question
65-year-old West Texas farmer of Swedish ancestry has an indolent, pale, raised, waxy, 1.2-cm skin mass over the bridge of the nose. The mass has been slowly growing over the past 3 years. There are no enlarged lymph nodes in the neck. Other than a “weather-beaten” appearance for the rest of his exposed skin, the remainder of the physical examination is unremarkable. Which of the following is the most likely diagnosis?
Correct
The correct answer is A. Basal cell carcinoma affects sun-exposed areas, particularly the mid and upper face, in patients lacking protective pigmentation. One of its morphologic forms is that of a raised, waxy, pale lesion that grows very slowly and doesn’t metastasize to lymph nodes.
Melanoma (choice B) would have presented as a pigmented lesion, with asymmetry, irregular borders, different colors, and a diameter of more than 0.6 cm. It would not appear as a raised, waxy, pale nodule.
Keratoacanthoma (choice C) grows very rapidly in a matter of weeks and has a scaly, rough appearance, with a core of keratin.
If untreated, it eventually sloughs off. Pyogenic granuloma (choice D) also grows very rapidly. It has the appearance of wet granulation tissue, with visible yellowish pus.
Squamous cell carcinoma (choice E) is usually an ulcer, rather than a nodule. In the face, it favors the lower lip. If present for several years, lymph node metastasis can sometimes occur.Incorrect
The correct answer is A. Basal cell carcinoma affects sun-exposed areas, particularly the mid and upper face, in patients lacking protective pigmentation. One of its morphologic forms is that of a raised, waxy, pale lesion that grows very slowly and doesn’t metastasize to lymph nodes.
Melanoma (choice B) would have presented as a pigmented lesion, with asymmetry, irregular borders, different colors, and a diameter of more than 0.6 cm. It would not appear as a raised, waxy, pale nodule.
Keratoacanthoma (choice C) grows very rapidly in a matter of weeks and has a scaly, rough appearance, with a core of keratin.
If untreated, it eventually sloughs off. Pyogenic granuloma (choice D) also grows very rapidly. It has the appearance of wet granulation tissue, with visible yellowish pus.
Squamous cell carcinoma (choice E) is usually an ulcer, rather than a nodule. In the face, it favors the lower lip. If present for several years, lymph node metastasis can sometimes occur. -
Question 26 of 151
26. Question
A 49-year-old man presents to his physician for follow-up of a previous fasting total serum cholesterol level of 299 mg/dL. The patient is otherwise well, with only mild hypertension that he is attempting to control with exercise and a low-fat, low cholesterol diet. He has a family history of ischemic heart disease on his father’s side, and he smokes a pack of cigarettes daily. The patient is a plant manager and is happy with his job and reports no home-life issues with his wife or three sons. A lipid profile was drawn at his last visit, and he presents today for review of those results, which are as follows:
Total cholesterol 230 mg/dL
HDL 45 mg/dL
LDL 100 mg/dL
Which of the following is the most appropriate intervention at this time?Correct
The correct answer is C. This patient has two risk factors (family history and tobacco) but has an LDL less than 130 mg/dL and a total cholesterol less than 239 mg/dL. He can therefore be managed by diet and exercise with a follow up profile being drawn in 1 year.
No intervention is indicated (choice A) is incorrect.
The patient does have marginally elevated cholesterol and a suboptimal LDL component. Educating the patient about diet and exercise (choice B) is partially correct, but he must have follow up. Since the patient already adheres to a low-fat, low-cholesterol diet, initiating a mandatory low-fat diet (choice D) would be of little help.
Given the combination of this patient’s risk factors and cholesterol levels, there is no reason to initiate drug therapy to control his hyperlipidemia (choice E). His LDL would need to be greater than 130 mg/dL or his total cholesterol greater than 239 mg/dL to initiate drug therapy.Incorrect
The correct answer is C. This patient has two risk factors (family history and tobacco) but has an LDL less than 130 mg/dL and a total cholesterol less than 239 mg/dL. He can therefore be managed by diet and exercise with a follow up profile being drawn in 1 year.
No intervention is indicated (choice A) is incorrect.
The patient does have marginally elevated cholesterol and a suboptimal LDL component. Educating the patient about diet and exercise (choice B) is partially correct, but he must have follow up. Since the patient already adheres to a low-fat, low-cholesterol diet, initiating a mandatory low-fat diet (choice D) would be of little help.
Given the combination of this patient’s risk factors and cholesterol levels, there is no reason to initiate drug therapy to control his hyperlipidemia (choice E). His LDL would need to be greater than 130 mg/dL or his total cholesterol greater than 239 mg/dL to initiate drug therapy. -
Question 27 of 151
27. Question
A 30-year-old woman presents with low back pain, and a swollen and painful right ankle and left knee for 1 week. She also reports temperatures to 39.0 C (102.2 F). She recalls that a few weeks ago, she had diarrhea, abdominal cramps, and fever, which lasted for a few days. She does not have previous history of joint diseases or sexually transmitted diseases. Examination reveals skin pustules and crusted vesicles on her palms and soles, red conjunctivae, and arthritis of the left knee and right ankle. There is no hepatomegaly, splenomegaly, or lymphadenopathy. Laboratory analysis show an elevated erythrocyte sedimentation rate (ESR), but no anemia. Which of the following is the most likely diagnosis?
Correct
The correct answer is B. Reiter syndrome develops as a complication of two types of infections: enteric infections due to Shigella, Salmonella, Yersinia, or Campylobacter, and sexually transmitted diseases caused by Chlamydia or Ureaplasma. Eighty percent of cases occur in HLA-B27-positive individuals. A male predominance of 9:1 is characteristic of cases following sexually transmitted infections. Nonsteroidal anti-inflammatory drugs (NSAIDs) are used for treatment; antibiotics are used to reduce the likelihood of developing Reiter syndrome after sexually transmitted diseases, but not after enteric infections.
Psoriatic arthritis (choice A) is similar to rheumatoid arthritis with regard to the pattern of joint involvement, but is not associated with a positive rheumatoid factor. Usually, psoriasis antedates the onset of arthritis, but sometimes the opposite occurs. Involvement of sacroiliac joints is common in this condition. Arthritis associated with rheumatic fever (choice C) is characterized by a migratory course and is associated with signs and symptoms of cardiac involvement.
Skin lesions are known as erythema marginatum. Rheumatoid arthritis (choice D) characteristically affects small joints (most commonly in the hands) in a symmetric fashion. Most cases show circulating rheumatoid factor.
The manifestations of syphilitic infection (choice E) are classically divided into primary, secondary, and tertiary stages. The primary stage is characterized by a painless ulcer at the site of entry, ie, a chancre. The secondary stage develops up to 6 months following the primary stage and is associated with a diffuse maculopapular eruption involving skin and mucous membranes, generalized lymphadenopathy, fever, hepatitis, iritis, osteitis, aseptic meningitis, and arthritis. The tertiary stage manifests after long periods of latency and involves the ascending aorta, bones, liver, skin, and CNS.Incorrect
The correct answer is B. Reiter syndrome develops as a complication of two types of infections: enteric infections due to Shigella, Salmonella, Yersinia, or Campylobacter, and sexually transmitted diseases caused by Chlamydia or Ureaplasma. Eighty percent of cases occur in HLA-B27-positive individuals. A male predominance of 9:1 is characteristic of cases following sexually transmitted infections. Nonsteroidal anti-inflammatory drugs (NSAIDs) are used for treatment; antibiotics are used to reduce the likelihood of developing Reiter syndrome after sexually transmitted diseases, but not after enteric infections.
Psoriatic arthritis (choice A) is similar to rheumatoid arthritis with regard to the pattern of joint involvement, but is not associated with a positive rheumatoid factor. Usually, psoriasis antedates the onset of arthritis, but sometimes the opposite occurs. Involvement of sacroiliac joints is common in this condition. Arthritis associated with rheumatic fever (choice C) is characterized by a migratory course and is associated with signs and symptoms of cardiac involvement.
Skin lesions are known as erythema marginatum. Rheumatoid arthritis (choice D) characteristically affects small joints (most commonly in the hands) in a symmetric fashion. Most cases show circulating rheumatoid factor.
The manifestations of syphilitic infection (choice E) are classically divided into primary, secondary, and tertiary stages. The primary stage is characterized by a painless ulcer at the site of entry, ie, a chancre. The secondary stage develops up to 6 months following the primary stage and is associated with a diffuse maculopapular eruption involving skin and mucous membranes, generalized lymphadenopathy, fever, hepatitis, iritis, osteitis, aseptic meningitis, and arthritis. The tertiary stage manifests after long periods of latency and involves the ascending aorta, bones, liver, skin, and CNS. -
Question 28 of 151
28. Question
79-year-old man, weighing 75 kg, with emphysema is intubated in the intensive care unit because of respiratory failure after developing adult respiratory distress syndrome secondary to an Escherichia coli bacteremia after an untreated urinary tract infection. His ventilator is set to a respiratory rate of 20/min, a tidal volume of 750 mL/breath, and a pO2 of 100%. If these settings are continued for the next 72 hours, the patient is at most increased risk for the development of which of the following complications?
Correct
The correct answer is D. High concentrations of inspired oxygen delivered through a ventilator may lead to pulmonary fibrosis, which becomes irreversible. In the setting of adult respiratory distress syndrome (ARDS), if the inspired fraction of oxygen cannot be lowered without producing hypoxia, the addition of positive-end expiratory pressure (PEEP) is indicated. Although PEEP does increase the risk of both barotrauma and hypotension by impairing right-sided heart filling, it is indicated to prevent the development of oxygen toxicity, which may result in irreversible pulmonary fibrosis.
Congestive heart failure (choice A) can occur as a complication of the patient’s longstanding underlying pulmonary disease, but the incidence would not likely be increased because of his ventilator settings.
Jugular venous distension (choice B) can be a marker for either right heart congestive failure (see choice A discussion) or tension pneumothorax (see choice E discussion), but would not be a likely complication of a high pO2.
Pulmonary embolus (choice C) would more likely be related to prolonged bed rest with resultant venous thrombosis. Tension pneumothorax (choice E) would be more likely to occur if the tidal volume were significantly greater than 750 mL/breath (corresponding to the optimal flow rate of 10 mL/ kg).Incorrect
The correct answer is D. High concentrations of inspired oxygen delivered through a ventilator may lead to pulmonary fibrosis, which becomes irreversible. In the setting of adult respiratory distress syndrome (ARDS), if the inspired fraction of oxygen cannot be lowered without producing hypoxia, the addition of positive-end expiratory pressure (PEEP) is indicated. Although PEEP does increase the risk of both barotrauma and hypotension by impairing right-sided heart filling, it is indicated to prevent the development of oxygen toxicity, which may result in irreversible pulmonary fibrosis.
Congestive heart failure (choice A) can occur as a complication of the patient’s longstanding underlying pulmonary disease, but the incidence would not likely be increased because of his ventilator settings.
Jugular venous distension (choice B) can be a marker for either right heart congestive failure (see choice A discussion) or tension pneumothorax (see choice E discussion), but would not be a likely complication of a high pO2.
Pulmonary embolus (choice C) would more likely be related to prolonged bed rest with resultant venous thrombosis. Tension pneumothorax (choice E) would be more likely to occur if the tidal volume were significantly greater than 750 mL/breath (corresponding to the optimal flow rate of 10 mL/ kg). -
Question 29 of 151
29. Question
48-year-old man comes to the physician because of the recent onset of obesity and easy bruisability. His blood pressure is 165/95 mm Hg. Dermal striae are found on physical examination. A dipstick examination of urine reveals glycosuria. Which of the following is the most appropriate next step in diagnosis?
Correct
The correct answer is C. In the presence of hypertension of recent onset, along with dermal striae, easy bruisability, and evidence of glucose intolerance (glycosuria), hypercortisolism should be suspected. The first test to perform is the dexamethasone suppression, which consists of administering 1 mg of dexamethasone at 11 PM, and then measuring serum cortisol levels in a blood sample drawn at 8 AM next day. Abnormally high cortisol levels after this test confirms hypercortisolism. The next step is to find the source of excessive cortisol or ACTH. Other manifestations of hypercortisolism include osteoporosis, muscle wasting, psychologic alterations, hirsutism, and granulocytic leukocytosis with lymphopenia.
Baseline plasma ACTH measurement (choice A) should be performed after a diagnosis of hypercortisolism has been confirmed by dexamethasone suppression test. This test is used to determine whether excessive cortisol production is secondary to increased ACTH levels. CT scans of the chest and abdomen (choice B) are performed to look for ectopic sources of ACTH. These radiologic studies are especially useful in finding neoplasms that may manifest with inappropriate ACTH secretion, the most frequent of which being small cell carcinoma of the lungs.
Measurement of midnight serum cortisol level (choice D) helps distinguish Cushing syndrome from other causes of pseudo-Cushing states, such as alcoholism, depression, and anorexia nervosa. This test requires the patient to remain in the same time zone for 3 consecutive days, be without food for at least 3 hours, and have an indwelling catheter ready for the blood draw.
Measurement of 24-hour urine cortisol and creatinine (choice E) may become necessary for some patients in whom the dexamethasone suppression test gives equivocal results. The above mentioned pseudo-Cushing states, however, may show abnormally high levels of free urine cortisol.
MRI of the head (choice F) is the method of choice to study the pituitary gland and look for adenomas. ACTH-producing adenoma is the cause of Cushing disease, which accounts for 70% of cases of hypercortisolism not due to exogenous corticosteroids.Incorrect
The correct answer is C. In the presence of hypertension of recent onset, along with dermal striae, easy bruisability, and evidence of glucose intolerance (glycosuria), hypercortisolism should be suspected. The first test to perform is the dexamethasone suppression, which consists of administering 1 mg of dexamethasone at 11 PM, and then measuring serum cortisol levels in a blood sample drawn at 8 AM next day. Abnormally high cortisol levels after this test confirms hypercortisolism. The next step is to find the source of excessive cortisol or ACTH. Other manifestations of hypercortisolism include osteoporosis, muscle wasting, psychologic alterations, hirsutism, and granulocytic leukocytosis with lymphopenia.
Baseline plasma ACTH measurement (choice A) should be performed after a diagnosis of hypercortisolism has been confirmed by dexamethasone suppression test. This test is used to determine whether excessive cortisol production is secondary to increased ACTH levels. CT scans of the chest and abdomen (choice B) are performed to look for ectopic sources of ACTH. These radiologic studies are especially useful in finding neoplasms that may manifest with inappropriate ACTH secretion, the most frequent of which being small cell carcinoma of the lungs.
Measurement of midnight serum cortisol level (choice D) helps distinguish Cushing syndrome from other causes of pseudo-Cushing states, such as alcoholism, depression, and anorexia nervosa. This test requires the patient to remain in the same time zone for 3 consecutive days, be without food for at least 3 hours, and have an indwelling catheter ready for the blood draw.
Measurement of 24-hour urine cortisol and creatinine (choice E) may become necessary for some patients in whom the dexamethasone suppression test gives equivocal results. The above mentioned pseudo-Cushing states, however, may show abnormally high levels of free urine cortisol.
MRI of the head (choice F) is the method of choice to study the pituitary gland and look for adenomas. ACTH-producing adenoma is the cause of Cushing disease, which accounts for 70% of cases of hypercortisolism not due to exogenous corticosteroids. -
Question 30 of 151
30. Question
A 4-year-old boy is brought to the emergency department for a painful and swollen right forearm. He was bitten and scratched by a family cat 2 days ago in the affected area. His temperature is 39.6 C (103.2 F). The right forearm is erythematous, edematous, and tender to touch. Which of the following is the most appropriate antibiotic treatment for this patient?
Correct
The correct answer is B. Animal bite is a common problem in pediatrics. In this case, the patient was bitten by a cat. To initiate appropriate antibiotic treatment, one needs to understand which organisms are most likely causing this infection. In a cat-bite wound, the most common organisms isolated are Pasteurella multocida and Staphylococcus aureus. Among the choices, amoxicillin-clavulanate is the only antibiotic that is effective against both organisms. P. multocida infection usually manifests within 24-48 hours following the bite or scratch as localized swelling, erythema, tenderness, and serous or sanguinopurulent discharge. Fever, chills, and lymphadenopathy can also occur. Complications include tenosynovitis, osteomyelitis, and septic arthritis. P. multocida is found in the oral flora of 70% to 90% of cats, 25% to 50% of dogs, and a variable percentage in other animals. S. aureus is a common pathogen for cellulitis. It is often found on the skin. In a severe case of animal bite, the pathogens are usually polymicrobial. Therefore, amoxicillin-clavulanate is the treatment of choice; 7-10 days therapy is usually sufficient. Ampicillin (choice A) is an effective treatment against P. multocida, but it has no effect on S. aureus.
Clindamycin (choice C) is effective against most serotypes of S. aureus, but is ineffective against P. multocida.
Tetracycline (choice D) is effective again P. multocida, but it should not be given to children younger than 8 years.
Trimethoprim-sulfamethoxazole (choice E) is not adequate on its own. When combined with clindamycin, however, it is an alternative treatment if the patient is allergic to penicillin compounds.Incorrect
The correct answer is B. Animal bite is a common problem in pediatrics. In this case, the patient was bitten by a cat. To initiate appropriate antibiotic treatment, one needs to understand which organisms are most likely causing this infection. In a cat-bite wound, the most common organisms isolated are Pasteurella multocida and Staphylococcus aureus. Among the choices, amoxicillin-clavulanate is the only antibiotic that is effective against both organisms. P. multocida infection usually manifests within 24-48 hours following the bite or scratch as localized swelling, erythema, tenderness, and serous or sanguinopurulent discharge. Fever, chills, and lymphadenopathy can also occur. Complications include tenosynovitis, osteomyelitis, and septic arthritis. P. multocida is found in the oral flora of 70% to 90% of cats, 25% to 50% of dogs, and a variable percentage in other animals. S. aureus is a common pathogen for cellulitis. It is often found on the skin. In a severe case of animal bite, the pathogens are usually polymicrobial. Therefore, amoxicillin-clavulanate is the treatment of choice; 7-10 days therapy is usually sufficient. Ampicillin (choice A) is an effective treatment against P. multocida, but it has no effect on S. aureus.
Clindamycin (choice C) is effective against most serotypes of S. aureus, but is ineffective against P. multocida.
Tetracycline (choice D) is effective again P. multocida, but it should not be given to children younger than 8 years.
Trimethoprim-sulfamethoxazole (choice E) is not adequate on its own. When combined with clindamycin, however, it is an alternative treatment if the patient is allergic to penicillin compounds. -
Question 31 of 151
31. Question
A 1-month old boy is brought to the emergency department by his mother, who states that he has been having what she describes as “projectile vomiting” for the past several days. She states that he vomits every time she feeds him, and the situation seems to be getting worse, although he does not seem to be in pain. She describes the vomitus as non-bilious, and he has had normal stools with no blood in them. On examination, the infant appears to be mildly dehydrated, his abdomen is soft, and there is a palpable, olive-sized, firm moveable mass in the right upper quadrant. Which of the following is the most likely diagnosis?
Correct
The correct answer is E. Pyloric stenosis develops in the first weeks of life. It is caused by hypertrophy of the pyloric muscle, which obstructs gastric outflow. The incidence is higher in males and first-born infants. The symptoms include progressively worsening vomiting, which becomes projectile and is non-bilious. On examination, peristaltic waves may be seen, and an olive-sized mass is usually palpated in the right upper quadrant. Duodenal atresia (choice A) is usually associated with other congenital anomalies. Symptoms include bilious vomiting, abdominal distention, and failure to pass meconium. Upright abdominal x-ray films show the classic “double-bubble sign.”
Intussusception (choice B) presents with vomiting, bloody stool, and colicky abdominal pain. As the obstruction progresses, the vomitus becomes bile-stained. On examination, there is usually a sausage-shaped mass in the distribution of the colon.
Hirschsprung disease (choice C), or aganglionic megacolon, is associated with failure to pass meconium or constipation and abdominal distention. Diagnosis is made by rectal biopsy.
Midgut volvulus (choice D) can occur at any age but is common in infancy. Symptoms include bilious vomiting, abdominal distention, pain, and bloody stools. An upper gastrointestinal series is diagnostic for volvulus showing a “corkscrew” narrowing of the distal duodenum.Incorrect
The correct answer is E. Pyloric stenosis develops in the first weeks of life. It is caused by hypertrophy of the pyloric muscle, which obstructs gastric outflow. The incidence is higher in males and first-born infants. The symptoms include progressively worsening vomiting, which becomes projectile and is non-bilious. On examination, peristaltic waves may be seen, and an olive-sized mass is usually palpated in the right upper quadrant. Duodenal atresia (choice A) is usually associated with other congenital anomalies. Symptoms include bilious vomiting, abdominal distention, and failure to pass meconium. Upright abdominal x-ray films show the classic “double-bubble sign.”
Intussusception (choice B) presents with vomiting, bloody stool, and colicky abdominal pain. As the obstruction progresses, the vomitus becomes bile-stained. On examination, there is usually a sausage-shaped mass in the distribution of the colon.
Hirschsprung disease (choice C), or aganglionic megacolon, is associated with failure to pass meconium or constipation and abdominal distention. Diagnosis is made by rectal biopsy.
Midgut volvulus (choice D) can occur at any age but is common in infancy. Symptoms include bilious vomiting, abdominal distention, pain, and bloody stools. An upper gastrointestinal series is diagnostic for volvulus showing a “corkscrew” narrowing of the distal duodenum. -
Question 32 of 151
32. Question
A 75-year-old man has widely disseminated prostate cancer. He is being followed at home because he has refused any further specific anti-cancer therapy. His hematocrit drops from a previously stable 42% to 25% over a two month period. Review of the peripheral blood smear demonstrates normochromia, anisocytosis, poikilocytosis, and nucleated red blood cells. Rare immature myeloid cells are also seen in the smear. This patient’s anemia is best classified as which of the following?
Correct
The correct answer is C. Myelophthisic anemia is anemia caused by replacement of marrow by other constituents, either abnormal hematopoietic cells or nonhematopoietic cells. These, in effect, “push out” the normal marrow constituents leading to the anemia. The fragmented and distorted cells seen in anemias due to this type of process are the result of trauma to erythrocytes going through distorted and compressed capillary beds. If the underlying disease process is (not very common) myelofibrosis or a (rare) hereditary storage disease, splenomegaly (sometimes massive) related to either extramedullary hematopoiesis or underlying disease process may be present. When the underlying disease process is an end-stage cancer (common) or granulomatous disease (less common), splenomegaly is usually not seen because little extramedullary hematopoiesis occurs. Treatment involves management of the underlying disease, if possible, with transfusions indicated if the anemia produces cardiovascular symptoms. Erythropoietin is often tried in these settings, but typically produces only a modest response since the underlying problem is a lack of marrow room rather than stagnant erythrocyte development.
Autoimmune anemias (choice A) tend to be hemolytic and tend to have normal appearing erythrocytes on peripheral smear; autoagglutination of red cells may be seen in some types.
Hereditary hemoglobin synthesis problems (choice B) include diseases such as sickle cell anemia, thalassemia, and hemoglobin C disease, and have usually been apparent through most of the patient’s life.
Nutritional deficiency anemias (choice D) can be either megaloblastic (folate, vitamin B12, or less commonly vitamin C deficiency) or microcytic (iron deficiency). Milder forms can sometimes be normocytic. Fragmented and mechanically distorted erythrocytes are not seen in these processes.
Anemias due to red cell membrane defects (choice E) include hereditary spherocytosis and hereditary elliptocytosis. These disorders produce, respectively, regular spherocytic and regular elliptical erythrocytes, but would not produce fragmented and mechanically distorted erythrocytes.Incorrect
The correct answer is C. Myelophthisic anemia is anemia caused by replacement of marrow by other constituents, either abnormal hematopoietic cells or nonhematopoietic cells. These, in effect, “push out” the normal marrow constituents leading to the anemia. The fragmented and distorted cells seen in anemias due to this type of process are the result of trauma to erythrocytes going through distorted and compressed capillary beds. If the underlying disease process is (not very common) myelofibrosis or a (rare) hereditary storage disease, splenomegaly (sometimes massive) related to either extramedullary hematopoiesis or underlying disease process may be present. When the underlying disease process is an end-stage cancer (common) or granulomatous disease (less common), splenomegaly is usually not seen because little extramedullary hematopoiesis occurs. Treatment involves management of the underlying disease, if possible, with transfusions indicated if the anemia produces cardiovascular symptoms. Erythropoietin is often tried in these settings, but typically produces only a modest response since the underlying problem is a lack of marrow room rather than stagnant erythrocyte development.
Autoimmune anemias (choice A) tend to be hemolytic and tend to have normal appearing erythrocytes on peripheral smear; autoagglutination of red cells may be seen in some types.
Hereditary hemoglobin synthesis problems (choice B) include diseases such as sickle cell anemia, thalassemia, and hemoglobin C disease, and have usually been apparent through most of the patient’s life.
Nutritional deficiency anemias (choice D) can be either megaloblastic (folate, vitamin B12, or less commonly vitamin C deficiency) or microcytic (iron deficiency). Milder forms can sometimes be normocytic. Fragmented and mechanically distorted erythrocytes are not seen in these processes.
Anemias due to red cell membrane defects (choice E) include hereditary spherocytosis and hereditary elliptocytosis. These disorders produce, respectively, regular spherocytic and regular elliptical erythrocytes, but would not produce fragmented and mechanically distorted erythrocytes. -
Question 33 of 151
33. Question
A 9-year-old boy is brought to the pediatrician’s office for bed-wetting. His mother states that he has never been dry at night. Occasionally, he has problems controlling his bladder during the day. On physical examination, his blood pressure is 98/56 mm Hg. Both his weight and height are below the 5th percentile for his age. His bladder is enlarged and palpable above the symphysis pubis. Which of the following is the most likely cause of his problem?
Correct
The correct answer is A. These findings suggest that this boy has bladder outlet obstruction. The most common cause of urethral obstruction in males is posterior urethral valves. It typically results in urinary obstruction and vesicoureteral reflux. Its incidence is 1 in 5000-8000 males. The degree of obstruction varies; therefore, the degree of renal failure also varies. It can range from mild hydronephrosis to renal dysplasia. Voiding cystourethrogram is the definitive diagnostic test. Urethral strictures and urethral meatal stenosis can also cause obstruction distal to the urinary bladder, but these conditions are uncommon.
Primary polydipsia (choice B) can cause enuresis, but growth failure is not characteristic.
Reflux nephropathy (choice C) occurs in patients who have vesicoureteral reflux. The bladder should not be palpable.
Sickle cell trait (choice D) can cause hyposthenuria, but it usually happens in African American boys in late childhood or adulthood.
An unstable bladder (choice E) is not palpable, and growth is normal.Incorrect
The correct answer is A. These findings suggest that this boy has bladder outlet obstruction. The most common cause of urethral obstruction in males is posterior urethral valves. It typically results in urinary obstruction and vesicoureteral reflux. Its incidence is 1 in 5000-8000 males. The degree of obstruction varies; therefore, the degree of renal failure also varies. It can range from mild hydronephrosis to renal dysplasia. Voiding cystourethrogram is the definitive diagnostic test. Urethral strictures and urethral meatal stenosis can also cause obstruction distal to the urinary bladder, but these conditions are uncommon.
Primary polydipsia (choice B) can cause enuresis, but growth failure is not characteristic.
Reflux nephropathy (choice C) occurs in patients who have vesicoureteral reflux. The bladder should not be palpable.
Sickle cell trait (choice D) can cause hyposthenuria, but it usually happens in African American boys in late childhood or adulthood.
An unstable bladder (choice E) is not palpable, and growth is normal. -
Question 34 of 151
34. Question
9-year-old boy is brought to the pediatric clinic by his mother, who noticed that the left side of his mouth has started to droop over the past several days. In addition, he is unable to close his left eye completely and complains of it burning. Review of systems reveals a cold approximately two weeks ago and recent decreased taste sensation. Physical examination reveals a well nourished male with normal vital signs. There is left eye ptosis and mild erythema of the left conjunctiva. His smile is asymmetrical on the left. Laboratory evaluation, including a complete blood count and chemistry profile, are normal. Which of the following infections is most closely associated with this patient’s condition?
Correct
The correct answer is A. This patient has Bell palsy, a postinfectious allergic or immune demyelinating facial neuritis. Epstein-Barr virus is the preceding infection in approximately 20% of cases. 85% of patients have their symptoms resolve on their own over a period of several weeks. 10% retain mild facial weakness and 5% have permanent severe facial weakness. Therapeutic intervention should include daily and nocturnal eye lubricants to protect the cornea from drying.
Group A Streptococcus(choice B), HIV (choice C), and influenza (choice D) are not associated with Bell palsy. However, herpes simplex virus, Lyme disease caused by Borrelia burgdorferi and mumps have been associated with Bell palsy.
Measles (choice E) is associated with subacute sclerosing panencephalitis, a chronic encephalitis of the central nervous system manifested by progressively bizarre behavior and decline in cognitive function.Incorrect
The correct answer is A. This patient has Bell palsy, a postinfectious allergic or immune demyelinating facial neuritis. Epstein-Barr virus is the preceding infection in approximately 20% of cases. 85% of patients have their symptoms resolve on their own over a period of several weeks. 10% retain mild facial weakness and 5% have permanent severe facial weakness. Therapeutic intervention should include daily and nocturnal eye lubricants to protect the cornea from drying.
Group A Streptococcus(choice B), HIV (choice C), and influenza (choice D) are not associated with Bell palsy. However, herpes simplex virus, Lyme disease caused by Borrelia burgdorferi and mumps have been associated with Bell palsy.
Measles (choice E) is associated with subacute sclerosing panencephalitis, a chronic encephalitis of the central nervous system manifested by progressively bizarre behavior and decline in cognitive function. -
Question 35 of 151
35. Question
A 4-year-old girl is brought to the physician because of a crusted honey-colored erythema resulting from rupture of tiny vesicles and pustules. Her temperature is 37.7 C (102 F). Skin lesions are distributed over the face and extremities. Physical examination reveals enlargement of lymph nodes in the cervical and axillary regions. Which of the following is the most frequent pathogen of this skin infection?
Correct
The correct answer is B. The presentation is consistent with impetigo. Impetigo may be of two forms, nonbullous and bullous. Nonbullous impetigo (more frequent) develops by colonization of traumatized skin by pyogenic bacteria and manifests with the characteristic vesiculopustular eruption that results in honey-colored crusts. Bullous impetigo is due to a strain of Staphylococcus aureus that produces exfoliatin, a toxin causing intraepidermal cleavage. Bullous impetigo manifests with large flaccid bullae that rupture easily.
Currently, Staphylococcus aureus is the etiologic agent of most cases of nonbullous impetigo, while until two decades ago Streptococcus pyogenes(choice C) accounted for the majority of cases. This change has important therapeutic implications. S. pyogenes is sensitive to penicillin, while S. aureus is not. Thus, the most appropriate treatment of both forms of impetigo is based on antibiotics effective against S. aureus. Treatment options include systemic erythromycin, first-generation cephalosporins, and topical mupirocin. Mupirocin is a fermentation product deriving from Pseudomonas fluorescens. It acts by inhibiting bacterial protein synthesis. Human herpesvirus 7 (choice A) is suspected to be the etiologic agent of pytiriasis rosea. This self-limited dermatosis is characterized by scaly pink macules that affect young adults and children.
Propionibacterium acnes(choice D) is thought to play an important role in the pathogenesis of acne. Colonization by this bacterium leads to release of pro-inflammatory and chemotactic agents that induce inflammation of the pilosebaceous unit.
Trychophyton fungi (choice E) are the most common pathogenic dermatophytes causing tinea corporis, due to superficial invasion of the stratum corneum by fungal organisms.Incorrect
The correct answer is B. The presentation is consistent with impetigo. Impetigo may be of two forms, nonbullous and bullous. Nonbullous impetigo (more frequent) develops by colonization of traumatized skin by pyogenic bacteria and manifests with the characteristic vesiculopustular eruption that results in honey-colored crusts. Bullous impetigo is due to a strain of Staphylococcus aureus that produces exfoliatin, a toxin causing intraepidermal cleavage. Bullous impetigo manifests with large flaccid bullae that rupture easily.
Currently, Staphylococcus aureus is the etiologic agent of most cases of nonbullous impetigo, while until two decades ago Streptococcus pyogenes(choice C) accounted for the majority of cases. This change has important therapeutic implications. S. pyogenes is sensitive to penicillin, while S. aureus is not. Thus, the most appropriate treatment of both forms of impetigo is based on antibiotics effective against S. aureus. Treatment options include systemic erythromycin, first-generation cephalosporins, and topical mupirocin. Mupirocin is a fermentation product deriving from Pseudomonas fluorescens. It acts by inhibiting bacterial protein synthesis. Human herpesvirus 7 (choice A) is suspected to be the etiologic agent of pytiriasis rosea. This self-limited dermatosis is characterized by scaly pink macules that affect young adults and children.
Propionibacterium acnes(choice D) is thought to play an important role in the pathogenesis of acne. Colonization by this bacterium leads to release of pro-inflammatory and chemotactic agents that induce inflammation of the pilosebaceous unit.
Trychophyton fungi (choice E) are the most common pathogenic dermatophytes causing tinea corporis, due to superficial invasion of the stratum corneum by fungal organisms. -
Question 36 of 151
36. Question
A 62-year-old man with a 110 pack-year history of smoking presents with chest pain. He states that for the past few months, he has been getting chest “pressure” localized to the substernal region, radiating to the left arm on occasion. The pain occurs with mild exertion, but never at rest. He further states that when he gets the pain, it usually last about 5 minutes but goes away with rest. He reports that his exercise tolerance is moderate, and he gets dyspnea on exertion after a few blocks of walking. On physical examination, he has no chest wall tenderness to palpation, but a carotid bruit is heard, and his dorsalis pedis pulses are decreased. He has no history of coronary disease but his family history is significant for his father having a myocardial infarction at age 56. He denies chest pain at this time. In addition to ascertaining his other coronary risk factors, which of the following is the most appropriate diagnostic intervention?
Correct
The correct answer is C. This is a patient who has 3 clear risk factors for coronary artery disease (tobacco, family history and age) and based on his physical examination, likely has severe peripheral vascular disease. He has, by definition, typical chest pain, so called “new onset angina”. He is a prime patient to have significant coronary disease, and thus we suspect ischemia as a cause for his pain. As a surrogate for coronary angiography, which actually shows anatomy, an exercise treadmill test allows us to detect ECG changes of ischemia with activity and thus stratify this patient as requiring intervention (such as percutaneous transluminal coronary angioplasty, or coronary artery bypass grafts), or perhaps angiography to better evaluate his anatomy.
A resting ECG (choice A) is appropriate, but not the most appropriate, given that he is pain-free at present and one would not expect to see any ECG changes associated with ischemia.
A cardiac echocardiogram (choice B) will likely be performed, given his dyspnea on exertion, but is not an appropriate test in the triaging of suspected ischemic chest pain. In some centers a “stress-echo”, specifically a dobutamine echocardiogram, is used to evaluate ischemic potential.
A non-urgent coronary angiography (choice D) is also inappropriate since angiography is an invasive procedure reserved for people that have had equivocal results from less invasive diagnostic procedures, or are having signs of crescendo angina. This patient has new angina, but it is “typical” angina in that it is exertional. An immediate coronary angiogram (choice E) is clearly not indicated as the patient is not having active ischemia or a myocardial infarction requiring reperfusion.Incorrect
The correct answer is C. This is a patient who has 3 clear risk factors for coronary artery disease (tobacco, family history and age) and based on his physical examination, likely has severe peripheral vascular disease. He has, by definition, typical chest pain, so called “new onset angina”. He is a prime patient to have significant coronary disease, and thus we suspect ischemia as a cause for his pain. As a surrogate for coronary angiography, which actually shows anatomy, an exercise treadmill test allows us to detect ECG changes of ischemia with activity and thus stratify this patient as requiring intervention (such as percutaneous transluminal coronary angioplasty, or coronary artery bypass grafts), or perhaps angiography to better evaluate his anatomy.
A resting ECG (choice A) is appropriate, but not the most appropriate, given that he is pain-free at present and one would not expect to see any ECG changes associated with ischemia.
A cardiac echocardiogram (choice B) will likely be performed, given his dyspnea on exertion, but is not an appropriate test in the triaging of suspected ischemic chest pain. In some centers a “stress-echo”, specifically a dobutamine echocardiogram, is used to evaluate ischemic potential.
A non-urgent coronary angiography (choice D) is also inappropriate since angiography is an invasive procedure reserved for people that have had equivocal results from less invasive diagnostic procedures, or are having signs of crescendo angina. This patient has new angina, but it is “typical” angina in that it is exertional. An immediate coronary angiogram (choice E) is clearly not indicated as the patient is not having active ischemia or a myocardial infarction requiring reperfusion. -
Question 37 of 151
37. Question
A 69-year-old retired police officer presents with complaints of excruciating right foot pain. The pain began approximately 6 hours ago, immediately on awaking. One night earlier, he had gone out for a steak dinner and drank half of a bottle of wine to celebrate his recent retirement. His past medical history is significant for hypertension, for which he takes hydrochlorothiazide. On physical examination, he is afebrile but appears acutely uncomfortable. There is swelling and tenderness in the right ankle and in the first right toe. Laboratory tests reveal a white blood cell count of 12,400/mm3 and a hematocrit of 39%. Serum electrolytes and liver function tests are normal. Uric acid is 4.1 mg/dL (normal 2.5-5.5 mg/dL). Which of the following will most likely be seen on examination of the joint arthrocentesis?
Correct
The correct answer is C. This patient has the acute onset of podagra and ankle involvement as well, secondary to gout. These attacks often follow the ingestion of red meats and/or alcohol since both of these lead to transient hyperuricemia. Note that this patient has a normal serum uric acid, which may occur during an acute attack; however, all patients with gout have some history of hyperuricemia. The appropriate evaluation of an edematous joint is an arthrocentesis. Examination of the fluid under polarizing light microscopy in a case such as this will reveal the typical uric acid crystals, which appear as negatively birefringent needle-shaped crystals (compare with choice E) under polarizing light microscopy.
Although it is always important to consider the possibility of an infectious arthritis, this patient has no risk factors for either gram-negative bacteremias or arthritis (choice A).
The gram-positive cocci in pairs and clusters (choice B) are describing Streptococcus, but this organism does not cause joint involvement.
Rhomboidal crystals (choice D), when positively birefringent, are due to calcium pyrophosphate (pseudogout crystals), and the typical history of recent ingestion of alcohol or red meat would not usually be elicited.Incorrect
The correct answer is C. This patient has the acute onset of podagra and ankle involvement as well, secondary to gout. These attacks often follow the ingestion of red meats and/or alcohol since both of these lead to transient hyperuricemia. Note that this patient has a normal serum uric acid, which may occur during an acute attack; however, all patients with gout have some history of hyperuricemia. The appropriate evaluation of an edematous joint is an arthrocentesis. Examination of the fluid under polarizing light microscopy in a case such as this will reveal the typical uric acid crystals, which appear as negatively birefringent needle-shaped crystals (compare with choice E) under polarizing light microscopy.
Although it is always important to consider the possibility of an infectious arthritis, this patient has no risk factors for either gram-negative bacteremias or arthritis (choice A).
The gram-positive cocci in pairs and clusters (choice B) are describing Streptococcus, but this organism does not cause joint involvement.
Rhomboidal crystals (choice D), when positively birefringent, are due to calcium pyrophosphate (pseudogout crystals), and the typical history of recent ingestion of alcohol or red meat would not usually be elicited. -
Question 38 of 151
38. Question
A 71-year-old woman is being treated for a severe chronic obstructive pulmonary disease (COPD) flare. The patient presented to the hospital 3 days ago with cough, fever, and pleuritic chest pain. She had been feeling fatigued and, on the day of admission, was persistently febrile. She has had more severe shortness of breath, often at rest and with mild exertion, and moderate dyspnea. A chest radiograph revealed a left lower lobe infiltrate, and she was started on antibiotics. The patient has a long smoking history with a forced expiration in 1 second (FEV1) of 1.1 L. Which of the following therapies is most beneficial with respect to long-term morbidity and mortality in this patient?
Correct
The correct answer is C. Two cardinal studies have shown a clear mortality benefit for hypoxic patients with chronic obstructive pulmonary disease (COPD) when they use long-term oxygen therapy (LTOT). These studies have shown that LTOT can increase body weight, alleviate cor pulmonale, decrease polycythemia, strengthen cardiac function, and improve exercise tolerance. Beta agonists (choice A) are widely prescribed as inhalers on an as-needed basis, but there is no evidence that this therapy is beneficial in any regard other than symptom relief.
Although there has been moderate debate about the use of systemic steroids, the use of inhaled corticosteroids (choice B) is fraught with even more issues and disagreement. Needless to say, even though these agents are used in the treatment of COPD patients, no data exist as to their long-term benefits or harm.
There has been a long-standing debate about the utility of systemic corticosteroids (choice D). It is now clear, however, that these agents have a beneficial effect in shortening the duration of COPD flares, but not in the therapy of stable disease.
Theophylline (choice E) has been used in the treatment of reversible airway obstruction (asthma) and COPD for a number of years. Recently, it has again become a useful agent in the therapy of COPD, in both stable and flare attacks. Its effect on long-term morbidity and mortality has yet to be documented, however.Incorrect
The correct answer is C. Two cardinal studies have shown a clear mortality benefit for hypoxic patients with chronic obstructive pulmonary disease (COPD) when they use long-term oxygen therapy (LTOT). These studies have shown that LTOT can increase body weight, alleviate cor pulmonale, decrease polycythemia, strengthen cardiac function, and improve exercise tolerance. Beta agonists (choice A) are widely prescribed as inhalers on an as-needed basis, but there is no evidence that this therapy is beneficial in any regard other than symptom relief.
Although there has been moderate debate about the use of systemic steroids, the use of inhaled corticosteroids (choice B) is fraught with even more issues and disagreement. Needless to say, even though these agents are used in the treatment of COPD patients, no data exist as to their long-term benefits or harm.
There has been a long-standing debate about the utility of systemic corticosteroids (choice D). It is now clear, however, that these agents have a beneficial effect in shortening the duration of COPD flares, but not in the therapy of stable disease.
Theophylline (choice E) has been used in the treatment of reversible airway obstruction (asthma) and COPD for a number of years. Recently, it has again become a useful agent in the therapy of COPD, in both stable and flare attacks. Its effect on long-term morbidity and mortality has yet to be documented, however. -
Question 39 of 151
39. Question
60-year-old woman complains of a 1-year history of bone pains, anorexia, weight loss, and constipation. Serum calcium is elevated and she is diagnosed with primary hyperparathyroidism. She is treated surgically, with removal of her parathyroid adenoma. She then has a long period of hypocalcemia and requires continuous treatment with vitamin D and calcium. Subsequently, after 6 months she becomes normocalcemic and does not need therapy. Which of the following conditions best explains these events?
Correct
The correct answer is C. The patient initially had osteitis fibrosa cystica as a result of the primary hyperparathyroidism. When the adenoma was removed and PTH level decreased, the skeleton underwent rapid remineralization. This created an increase in calcium requirement. Once the repair was complete, the calcium demand decreased. If the remaining glands were destroyed (choice A) she would have been permanently hypocalcemic.
In pseudohypoparathyroidism (choice B), there is end organ resistance to PTH, resulting in kidneys and bones being unresponsive. Patients have hypocalcemia and hyperphosphatemia.
Severe pancreatitis would cause saponification and hypocalcemia (choice D).
She would also have symptoms of epigastric pain. If the wrong gland were removed, she would be still hypercalcemic (choice E).Incorrect
The correct answer is C. The patient initially had osteitis fibrosa cystica as a result of the primary hyperparathyroidism. When the adenoma was removed and PTH level decreased, the skeleton underwent rapid remineralization. This created an increase in calcium requirement. Once the repair was complete, the calcium demand decreased. If the remaining glands were destroyed (choice A) she would have been permanently hypocalcemic.
In pseudohypoparathyroidism (choice B), there is end organ resistance to PTH, resulting in kidneys and bones being unresponsive. Patients have hypocalcemia and hyperphosphatemia.
Severe pancreatitis would cause saponification and hypocalcemia (choice D).
She would also have symptoms of epigastric pain. If the wrong gland were removed, she would be still hypercalcemic (choice E). -
Question 40 of 151
40. Question
A 6-month-old infant presents to the emergency department with the new onset of weak cry, decreased activity, and poor feeding. The mother also states that the infant has been constipated for the past 2 days. On physical examination, the infant has a very weak cry, poor muscle tone, and absent deep tendon reflexes. Which of the following is the most likely diagnosis?
Correct
The correct answer is C. Infant botulism results from the production of toxin after colonization of the gastrointestinal tract by Clostridium botulinum in young children aged 1-9 months. The most common source of the organism is the soil or, less frequently, honey. Nearly all cases are due to types A or B. The incubation period is usually between 18 and 36 hours. Short incubation periods are associated with more severe disease. The disease spectrum varies considerably, but the most commonly recognized form is the “floppy baby syndrome.” Initial symptoms are lethargy, diminished suck, constipation, weakness, feeble cry, and diminished spontaneous activity with loss of head control. These symptoms are followed by extensive flaccid paralysis. The case fatality rate is only 1%. The bulbar musculature is usually affected first. In older children, it results in diplopia, dysarthria, and dysphagia. Involvement of the cholinergic autonomic nervous system may result in decreased salivation, with dry mouth and sore throat, ileus, or urinary retention. Neurologic evaluation often shows bilateral paresis of the 6th cranial nerves, ptosis, dilated pupils with sluggish reaction, decreased gag reflex, or medial rectus paresis. These symptoms are followed by descending involvement of motor neurons to peripheral muscles, including the muscles of respiration. Patients are usually afebrile with clear mentation. The most common cause of death is respiratory failure. The spectrum of disease is quite variable; some patients have mild disease, whereas others have severe paralysis requiring mechanical ventilation. Respiratory failure is the major risk, and patients must be monitored carefully with liberal use of ventilatory support. Toxins can be removed from the gastrointestinal tract with gastric lavage, cathartics, and enemas early in the course of disease. The trivalent antitoxin or type-specific antitoxin for types A, B, and E is usually given only to adults. Infants with botulism should not receive either antibiotics directed against C. botulinum or antitoxin, because most do extremely well with supportive care alone and it has been suggested that antibiotics may cause toxin release. Honey has been implicated as a vehicle for spores and should not be fed to infants younger than 1 year.
Congenital hypothyroidism (choice A), or cretinism, manifests as impaired development of the skeletal system and CNS. It is associated with severe mental retardation, short stature, coarse facial features, a protruding tongue, and umbilical hernia.
Guillain-Barré syndrome (choice B) presents with weakness that develops symmetrically over several days. The weakness typically occurs first in the legs and ascends with time to involve the muscles of the trunk, intercostals, upper extremity, and neck. Muscles innervated by cranial nerves are also involved. Respiratory paralysis can lead to death within hours to days.
Myasthenia gravis (choice D) is a disorder of the neuromuscular junction resulting in a pure motor syndrome characterized by weakness and fatigue, particularly of the extraocular, pharyngeal, facial, cervical, proximal limb, and respiratory musculature. Fifteen percent of infants born to myasthenic mothers have neonatal myasthenia gravis because of the transplacental passage of acetylcholine receptor antibodies.
The condition completely resolves in weeks to months. Vaccine-associated poliomyelitis (choice E) is exceedingly rare; only eight or nine cases are reported yearly. Most symptomatic cases have nonspecific manifestations of infection. Illness is biphasic, and paralysis occurs in the second phase. Paralytic disease occurs with rapid onset, involving the cranial nerves, arms, and legs.Incorrect
The correct answer is C. Infant botulism results from the production of toxin after colonization of the gastrointestinal tract by Clostridium botulinum in young children aged 1-9 months. The most common source of the organism is the soil or, less frequently, honey. Nearly all cases are due to types A or B. The incubation period is usually between 18 and 36 hours. Short incubation periods are associated with more severe disease. The disease spectrum varies considerably, but the most commonly recognized form is the “floppy baby syndrome.” Initial symptoms are lethargy, diminished suck, constipation, weakness, feeble cry, and diminished spontaneous activity with loss of head control. These symptoms are followed by extensive flaccid paralysis. The case fatality rate is only 1%. The bulbar musculature is usually affected first. In older children, it results in diplopia, dysarthria, and dysphagia. Involvement of the cholinergic autonomic nervous system may result in decreased salivation, with dry mouth and sore throat, ileus, or urinary retention. Neurologic evaluation often shows bilateral paresis of the 6th cranial nerves, ptosis, dilated pupils with sluggish reaction, decreased gag reflex, or medial rectus paresis. These symptoms are followed by descending involvement of motor neurons to peripheral muscles, including the muscles of respiration. Patients are usually afebrile with clear mentation. The most common cause of death is respiratory failure. The spectrum of disease is quite variable; some patients have mild disease, whereas others have severe paralysis requiring mechanical ventilation. Respiratory failure is the major risk, and patients must be monitored carefully with liberal use of ventilatory support. Toxins can be removed from the gastrointestinal tract with gastric lavage, cathartics, and enemas early in the course of disease. The trivalent antitoxin or type-specific antitoxin for types A, B, and E is usually given only to adults. Infants with botulism should not receive either antibiotics directed against C. botulinum or antitoxin, because most do extremely well with supportive care alone and it has been suggested that antibiotics may cause toxin release. Honey has been implicated as a vehicle for spores and should not be fed to infants younger than 1 year.
Congenital hypothyroidism (choice A), or cretinism, manifests as impaired development of the skeletal system and CNS. It is associated with severe mental retardation, short stature, coarse facial features, a protruding tongue, and umbilical hernia.
Guillain-Barré syndrome (choice B) presents with weakness that develops symmetrically over several days. The weakness typically occurs first in the legs and ascends with time to involve the muscles of the trunk, intercostals, upper extremity, and neck. Muscles innervated by cranial nerves are also involved. Respiratory paralysis can lead to death within hours to days.
Myasthenia gravis (choice D) is a disorder of the neuromuscular junction resulting in a pure motor syndrome characterized by weakness and fatigue, particularly of the extraocular, pharyngeal, facial, cervical, proximal limb, and respiratory musculature. Fifteen percent of infants born to myasthenic mothers have neonatal myasthenia gravis because of the transplacental passage of acetylcholine receptor antibodies.
The condition completely resolves in weeks to months. Vaccine-associated poliomyelitis (choice E) is exceedingly rare; only eight or nine cases are reported yearly. Most symptomatic cases have nonspecific manifestations of infection. Illness is biphasic, and paralysis occurs in the second phase. Paralytic disease occurs with rapid onset, involving the cranial nerves, arms, and legs. -
Question 41 of 151
41. Question
A 69-year-old man in the cardiothoracic intensive care unit is postoperative day 6 for a coronary artery bypass graft. He was initially extubated on postoperative day 1, but was then re-intubated the following day for aspiration pneumonia, which had worsened despite administration of IV penicillin and gentamicin. He is also receiving daily furosemide, aspirin, and subcutaneous heparin 5000 units. One hour ago, he passed a large volume of black tarry stool. His temperature is 38.9 C (102 F), blood pressure is 180/94 mm Hg, and pulse is 102/min. Loud rhonchi are heard bilaterally. He has a regular, rapid heart rhythm with a II/VI systolic murmur. His abdomen is soft with mild epigastric tenderness. No masses are palpable. Rectal examination reveals no masses, and melena is present. Laboratory studies show a leukocyte count of 14,400, a hemoglobin of 9.8 g/dL, and a hematocrit of 29%. Which of the following is the most likely source of this patient’s bleeding?
Correct
The correct answer is B. This scenario is the classic situation for the development of diffuse stress gastritis, which has resulted in a large volume, black tarry stool. Furthermore, the patient is on aspirin and heparin, which predispose to bleeding from stress gastritis.
Cecal arterial venous malformations (choice A) are a common cause of bleeding in the elderly, but generally present with lower gastrointestinal bleeding with either hematochezia or maroon stool.
Although erosive esophagitis (choice C) may also occur in the critically ill patient, it is not as common as diffuse stress gastritis.
There is no evidence that the patient has esophageal varices (choice D), and patients in this setting are not at particular risk for a single giant gastric ulcer (choice E).Incorrect
The correct answer is B. This scenario is the classic situation for the development of diffuse stress gastritis, which has resulted in a large volume, black tarry stool. Furthermore, the patient is on aspirin and heparin, which predispose to bleeding from stress gastritis.
Cecal arterial venous malformations (choice A) are a common cause of bleeding in the elderly, but generally present with lower gastrointestinal bleeding with either hematochezia or maroon stool.
Although erosive esophagitis (choice C) may also occur in the critically ill patient, it is not as common as diffuse stress gastritis.
There is no evidence that the patient has esophageal varices (choice D), and patients in this setting are not at particular risk for a single giant gastric ulcer (choice E). -
Question 42 of 151
42. Question
A new technique for screening patients for lymphomas has been developed. This involves taking blood from candidates, attaching antibodies to a unique antigen (CD33-test), and analyzing the blood through flow cytometry. Those showing positivity on the test with the presence of the antigen CD33-test are thought to have a lymphoma. This is further corroborated with a bone marrow biopsy. After the appropriate human subject ethics committee has approved this study, the study is undertaken in 100 patients. The results are as follows:
Which of the following is the new test’s sensitivity?Correct
The correct answer is D. The sensitivity of a test is the probability that results will be positive in patients who actually have the disease. It reflects the tests ability to accurately diagnose all cases of the disease. When data are arranged as above, the sensitivity will be a/(a+c) = 62.5%. The specificity of a test is the probability that the test result will be negative in those without the disease.
If the patient has the disease but has a negative test result the result is termed a false negative. It is d/(b+d) = 33.3% (choice A).
The positive predictive value of a test is the probability that an individual who gets a positive test result actually has the disease. It is a/(a+b) = 38.5% (choice B).
The negative predictive value (choice C) of a test is the probability that an individual who gets a negative test result does not have the disease.
It is d/(c+d) = 57.1%. The sensitivity would be 100% (choice E) if the screening test were as accurate as the gold standard (biopsy), and would become the new gold standard.Incorrect
The correct answer is D. The sensitivity of a test is the probability that results will be positive in patients who actually have the disease. It reflects the tests ability to accurately diagnose all cases of the disease. When data are arranged as above, the sensitivity will be a/(a+c) = 62.5%. The specificity of a test is the probability that the test result will be negative in those without the disease.
If the patient has the disease but has a negative test result the result is termed a false negative. It is d/(b+d) = 33.3% (choice A).
The positive predictive value of a test is the probability that an individual who gets a positive test result actually has the disease. It is a/(a+b) = 38.5% (choice B).
The negative predictive value (choice C) of a test is the probability that an individual who gets a negative test result does not have the disease.
It is d/(c+d) = 57.1%. The sensitivity would be 100% (choice E) if the screening test were as accurate as the gold standard (biopsy), and would become the new gold standard. -
Question 43 of 151
43. Question
A 2-year-old girl is brought to the clinic with headache, vomiting, and pallor. Her blood pressure is 130/80 mm Hg. On physical examination, she is noted to have aniridia and a large abdominal mass. Abdominal scanning reveals a poorly vascularized tumor in the upper pole of the right kidney. Which of the following is the most likely cause of this presentation?
Correct
The correct answer is A. The blood pressure of this child is significantly elevated for her age. The age, hypertension, abdominal mass, and aniridia suggest Wilms tumor, which arises because of the deletion involving chromosome 11.
Fragile X syndrome (choice B) is a common cause of mental retardation.
Translocation of chromosomes 9 and 21 (choice C) is called the Philadelphia chromosome and causes leukemia.
Trisomy 13 (choice D) causes severe birth defects, including CNS malformations, cleft lip, polydactyly, and mental retardation.
Turner syndrome (choice E) is a cause of short stature, infertility, and mild mental retardation.Incorrect
The correct answer is A. The blood pressure of this child is significantly elevated for her age. The age, hypertension, abdominal mass, and aniridia suggest Wilms tumor, which arises because of the deletion involving chromosome 11.
Fragile X syndrome (choice B) is a common cause of mental retardation.
Translocation of chromosomes 9 and 21 (choice C) is called the Philadelphia chromosome and causes leukemia.
Trisomy 13 (choice D) causes severe birth defects, including CNS malformations, cleft lip, polydactyly, and mental retardation.
Turner syndrome (choice E) is a cause of short stature, infertility, and mild mental retardation. -
Question 44 of 151
44. Question
A 7-year-old boy is referred by his school for psychiatric evaluation. The teachers have noticed that, in the past year, he has been unable to sustain attention in class, and has been fidgeting and talking to his peers during class. He seems unable to wait for others to finish speaking and keeps interrupting and blurting out answers before questions are completed. At home, his parents state that he is forgetful and losses things easily. Which of the following is the most likely diagnosis?
Correct
The correct answer is A. Attention deficit/hyperactivity disorder (ADHD) is characterized by impulsivity, hyperactivity, and inattention lasting at least 6 months. To make the diagnosis, the disorder must have started before age 7, and six signs each of inattention and impulsivity/hyperactivity need to be present.
In children, bipolar disorder (choice B) often presents as extremely irritable and explosive mood, with poor psychosocial functioning, decreased sleep, talkativeness, racing thoughts, and high energy.
Conduct disorder (choice C) is diagnosed when the child has had symptoms of aggression toward people and animals, destruction of property, deceitfulness or theft, and serious violation of rules. One symptom needs to last at least 6 months, even though the symptoms may have been present intermittently for a year.
Posttraumatic stress disorder (choice D) develops following trauma, and the child usually has symptoms of hypervigilance, increased arousal avoidance, and autonomic reactivity.
Rett syndrome (choice E) belongs to the pervasive developmental disorders mostly seen in girls. The child has normal development through the first 5 months of life. During the second year, however, she develops multiple deficits, including decreased head growth, decreased hand skills, social impairment, and impaired gait and trunk movements.Incorrect
The correct answer is A. Attention deficit/hyperactivity disorder (ADHD) is characterized by impulsivity, hyperactivity, and inattention lasting at least 6 months. To make the diagnosis, the disorder must have started before age 7, and six signs each of inattention and impulsivity/hyperactivity need to be present.
In children, bipolar disorder (choice B) often presents as extremely irritable and explosive mood, with poor psychosocial functioning, decreased sleep, talkativeness, racing thoughts, and high energy.
Conduct disorder (choice C) is diagnosed when the child has had symptoms of aggression toward people and animals, destruction of property, deceitfulness or theft, and serious violation of rules. One symptom needs to last at least 6 months, even though the symptoms may have been present intermittently for a year.
Posttraumatic stress disorder (choice D) develops following trauma, and the child usually has symptoms of hypervigilance, increased arousal avoidance, and autonomic reactivity.
Rett syndrome (choice E) belongs to the pervasive developmental disorders mostly seen in girls. The child has normal development through the first 5 months of life. During the second year, however, she develops multiple deficits, including decreased head growth, decreased hand skills, social impairment, and impaired gait and trunk movements. -
Question 45 of 151
45. Question
An elderly patient complains to a physician of sores in the mouth and on the skin. These lesions have developed over about a month. Physical examination demonstrates multiple painful erosions on the oral mucosa and tongue. Raw areas with crusting on the skin is seen on the face and trunk. The patient states that the skin lesions had started as blisters that had quickly broken. While the involved areas were painful, no itchiness had been experienced. Careful examination of the edge of the skin lesions demonstrates a few flaccid bullae. Rubbing of the skin near an affected area easily detaches the superficial part of the epidermis from the underlying skin (Nikolsky’s sign). No target-like lesions are seen. Which of the following is the most likely diagnosis?
Correct
The correct answer is C. The patient probably has pemphigus vulgaris, which is an uncommon autoimmune skin disorder characterized by blistering and erosions involving the mucous membranes and skin. The autoimmune attack is on the junctions between epithelial cells in the epidermis. The blisters occur high in the epithelium and can rupture easily, producing painful erosions. Nikolsky’s sign, in which rubbing of apparently unaffected skin causes a separation of the outer layers, is a helpful diagnostic clue. Pemphigus tends to begin in the mouth, where rapid rupture of the blisters may lead to the impression that the initial lesion is an ulcer rather than a blister. Biopsy with immunofluorescence studies can demonstrate blister formation high in the epithelium that is accompanied by IgG deposition on epithelial cell surfaces. Pemphigus vulgaris is a serious, chronic condition that can be life-threatening as a result of fluid/electrolyte imbalance, secondary infection, or complications of the high-dose corticosteroid therapy that may be necessary to bring the condition under control.
Bullous pemphigoid (choice A) causes tense bullae that do not rupture easily.
Dermatitis herpetiformis (choice B) causes clusters of intensely pruritic vesicles, papules, and urticarial lesions.
Stevens-Johnson syndrome (choice D) is a severe, blistering, form of erythema multiforme that characteristically shows at least a few target lesions.
Toxic epidermal necrolysis (choice E) can cause widespread flaccid blisters and Nikolsky’s sign may be positive, but this disorder tends to develop much more rapidly than pemphigus vulgaris.Incorrect
The correct answer is C. The patient probably has pemphigus vulgaris, which is an uncommon autoimmune skin disorder characterized by blistering and erosions involving the mucous membranes and skin. The autoimmune attack is on the junctions between epithelial cells in the epidermis. The blisters occur high in the epithelium and can rupture easily, producing painful erosions. Nikolsky’s sign, in which rubbing of apparently unaffected skin causes a separation of the outer layers, is a helpful diagnostic clue. Pemphigus tends to begin in the mouth, where rapid rupture of the blisters may lead to the impression that the initial lesion is an ulcer rather than a blister. Biopsy with immunofluorescence studies can demonstrate blister formation high in the epithelium that is accompanied by IgG deposition on epithelial cell surfaces. Pemphigus vulgaris is a serious, chronic condition that can be life-threatening as a result of fluid/electrolyte imbalance, secondary infection, or complications of the high-dose corticosteroid therapy that may be necessary to bring the condition under control.
Bullous pemphigoid (choice A) causes tense bullae that do not rupture easily.
Dermatitis herpetiformis (choice B) causes clusters of intensely pruritic vesicles, papules, and urticarial lesions.
Stevens-Johnson syndrome (choice D) is a severe, blistering, form of erythema multiforme that characteristically shows at least a few target lesions.
Toxic epidermal necrolysis (choice E) can cause widespread flaccid blisters and Nikolsky’s sign may be positive, but this disorder tends to develop much more rapidly than pemphigus vulgaris. -
Question 46 of 151
46. Question
A 57-year-old man presents to his physician for a preoperative evaluation. He has been a long-time patient in this office and has been treated for hypertension and gastritis. He has been scheduled for an elective open cholecystectomy in 2 days. He currently takes omeprazole for his gastritis and thiazide for his hypertension daily. He smokes two packs of cigarettes per day. His home blood pressure log shows that his systolic pressures range from 150 to 190 mm Hg, and his diastolic pressures range from 80 to 105 mm Hg, indicating that his blood pressure may be not adequately controlled for the surgical procedure. Which of the following medications is most appropriate in the perioperative period for added blood pressure control?
Correct
The correct answer is C. There is an extensive body of literature indicating that beta blockers given to non-cardiac surgical patients who are at risk of cardiac events are associated with a more favorable outcome in terms of postoperative cardiovascular morbidity and mortality. This patient has somewhat poorly controlled hypertension, as well as at least three cardiovascular risk factors (hypertension, tobacco, age). Ideally, one would like to have better control of the blood pressure and to reduce any risk for adverse perioperative events. Beta blockers can achieve both of these endpoints.
Captopril (choice A) is an ACE inhibitor that has good efficacy in the treatment of hypertension. This class of drugs has also been shown to prolong survival in patients with congestive heart failure.
Clonidine (choice B) is a central alpha-2 receptor agonist that works to attenuate sympathetic outflow and thus lower blood pressure. Although it is a reasonably efficacious drug, it is associated with rebound hypertension if abruptly discontinued. It has no role in the perioperative management of blood pressure.
Nifedipine (choice D) is a calcium channel blocker that has reasonable efficacy in treating hypertension. There is no benefit to giving this agent in the perioperative period.
Prazosin (choice E) is a alpha-1 receptor antagonist that is very efficacious in the treatment of hypertension. This class of drugs is also useful in the treatment of benign prostatic hypertrophy (BPH)Incorrect
The correct answer is C. There is an extensive body of literature indicating that beta blockers given to non-cardiac surgical patients who are at risk of cardiac events are associated with a more favorable outcome in terms of postoperative cardiovascular morbidity and mortality. This patient has somewhat poorly controlled hypertension, as well as at least three cardiovascular risk factors (hypertension, tobacco, age). Ideally, one would like to have better control of the blood pressure and to reduce any risk for adverse perioperative events. Beta blockers can achieve both of these endpoints.
Captopril (choice A) is an ACE inhibitor that has good efficacy in the treatment of hypertension. This class of drugs has also been shown to prolong survival in patients with congestive heart failure.
Clonidine (choice B) is a central alpha-2 receptor agonist that works to attenuate sympathetic outflow and thus lower blood pressure. Although it is a reasonably efficacious drug, it is associated with rebound hypertension if abruptly discontinued. It has no role in the perioperative management of blood pressure.
Nifedipine (choice D) is a calcium channel blocker that has reasonable efficacy in treating hypertension. There is no benefit to giving this agent in the perioperative period.
Prazosin (choice E) is a alpha-1 receptor antagonist that is very efficacious in the treatment of hypertension. This class of drugs is also useful in the treatment of benign prostatic hypertrophy (BPH) -
Question 47 of 151
47. Question
A 45-year-old woman comes to the physician because of progressive weakness for several months. At first, she became aware of increasing difficulty in climbing stairs, then lifting objects above head level. In the past week, the muscles of the shoulder girdle have become sore. She has had fever up to 38.5 C (101.3 F). On examination, proximal muscles of the upper and lower extremities are somewhat tender to palpation but not atrophic. There is periorbital edema associated with a purple discoloration of the upper eyelids. Scaly erythematous patches are seen on the dorsal aspect of hands and fingers. Laboratory studies show:
Hemoglobin…………………………..13.2g/dL
Leukocyte count………………………9300/mm3
Creatine kinase………………………2550 U/L
Erythrocyte sedimentation rate (ESR)……22mm/hr
Antinuclear antibody (ANA) titer……….Positive at 1:1280
Rheumatoid factor…………………….Positive at 1:512
A biopsy of the deltoid muscle shows atrophy of muscle fibers and interstitial lymphocytic infiltration. Which of the following is the most appropriate treatment at this time?Correct
The correct answer is E. The clinical picture is diagnostic of dermatomyositis. Women are more frequently affected than men. Muscle weakness and pain, along with the characteristic periorbital edema and purple discoloration of the upper eyelids, suggest the diagnosis, which is confirmed by muscle biopsy. The patches on the dorsal aspects of interphalangeal and metacarpophalangeal joints are referred to as Gottron sign. Anti-nuclear antibodies (ANAs) are found in the majority of patients, and rheumatoid factor is positive in a minority. Corticosteroids provide rapid relief in most patients. Prednisone is administered at high doses (40-60 mg daily).
Azathioprine (choice A) and methotrexate (choice B) are both effective, but they may cause serious adverse effects. Thus, they should be used only in patients intolerant or unresponsive to corticosteroids.
NSAIDs (choice C) are not sufficient to control the disease. Often, patients with dermatomyositis experience exacerbations when corticosteroids are withdrawn.
Plasma exchange (choice D), as well as leukapheresis, is not effective in treating dermatomyositis.Incorrect
The correct answer is E. The clinical picture is diagnostic of dermatomyositis. Women are more frequently affected than men. Muscle weakness and pain, along with the characteristic periorbital edema and purple discoloration of the upper eyelids, suggest the diagnosis, which is confirmed by muscle biopsy. The patches on the dorsal aspects of interphalangeal and metacarpophalangeal joints are referred to as Gottron sign. Anti-nuclear antibodies (ANAs) are found in the majority of patients, and rheumatoid factor is positive in a minority. Corticosteroids provide rapid relief in most patients. Prednisone is administered at high doses (40-60 mg daily).
Azathioprine (choice A) and methotrexate (choice B) are both effective, but they may cause serious adverse effects. Thus, they should be used only in patients intolerant or unresponsive to corticosteroids.
NSAIDs (choice C) are not sufficient to control the disease. Often, patients with dermatomyositis experience exacerbations when corticosteroids are withdrawn.
Plasma exchange (choice D), as well as leukapheresis, is not effective in treating dermatomyositis. -
Question 48 of 151
48. Question
A 34-year-old man presents with a swollen left knee of 2 days’ duration. He denies any known trauma to that region and has no prior history of any musculoskeletal complaints. He is in otherwise excellent health. He is homosexual and practices safe sex with a single partner. On physical examination, his knee is swollen, tender to palpation, and erythematous and has a limited range of motion. An arthrocentesis is performed. Which of the following is most suggestive of a septic arthritis in this patient?
Correct
The correct answer is E. Septic arthritis will produce the highest joint fluid white blood cell counts, typically with counts of greater than 75,000 per cc. Non-inflammatory arthritis, such as osteoarthritis, will typically produce joint aspirate counts of less than 10,000. If septic arthritis is suspected based upon the aspirate white blood cell count, then appropriate antibiotics should be started, while awaiting cultures. Failure to initiate appropriate antibiotic therapy until final identification of the organism would potentially lead to irreversible joint destruction. An elevated peripheral white blood cell count of 14,300 (choice A) may be seen in either a crystalline or septic arthritis and will not distinguish between the two.
Inflammatory arthritis, i.e. crystalline arthritis, will typically produce a joint fluid aspirate with a white blood cell count of approximately 25-50,000 (choices B, C, and D). Since the aspirated fluid is also routinely examined for crystals, differentiation of any cases of septic versus crystalline arthritis with borderline values for white blood cell count can usually be made on that basis.Incorrect
The correct answer is E. Septic arthritis will produce the highest joint fluid white blood cell counts, typically with counts of greater than 75,000 per cc. Non-inflammatory arthritis, such as osteoarthritis, will typically produce joint aspirate counts of less than 10,000. If septic arthritis is suspected based upon the aspirate white blood cell count, then appropriate antibiotics should be started, while awaiting cultures. Failure to initiate appropriate antibiotic therapy until final identification of the organism would potentially lead to irreversible joint destruction. An elevated peripheral white blood cell count of 14,300 (choice A) may be seen in either a crystalline or septic arthritis and will not distinguish between the two.
Inflammatory arthritis, i.e. crystalline arthritis, will typically produce a joint fluid aspirate with a white blood cell count of approximately 25-50,000 (choices B, C, and D). Since the aspirated fluid is also routinely examined for crystals, differentiation of any cases of septic versus crystalline arthritis with borderline values for white blood cell count can usually be made on that basis. -
Question 49 of 151
49. Question
An 8-year-old boy is seen in the pediatrician’s office for a routine health supervision visit. The mother states that he has Duchenne muscular dystrophy. On examination, he is found to have hip waddle and enlargement of both calves. He is ambulatory, but his muscle strength is diminished symmetrically. Which of the following signs is most consistent with Duchenne muscular dystrophy?
Correct
The correct answer is B. Duchenne muscular dystrophy is the most common type of muscular dystrophies. It is an X-linked genetic disorder and happens in 1:3600 boys. It is characterized by progressive muscle weakness caused by degeneration of muscle fibers. The disease is rarely symptomatic at birth, but usually develops to a clinically evident stage before the age of 5 years. Weakness is prominent in proximal muscles. Affected children often present with toe-walking and difficulty climbing stairs. The gait is waddling, and patients often fall. The Gower sign is very characteristic of Duchenne muscular dystrophy. It is considered positive if the patient uses his hands to “walk” up the legs when going from a prone to an upright sitting position because he does not have enough proximal muscle power to get up in a normal fashion. Pseudohypertrophy of the calves is also a prominent feature. Many patients will die in their 20s because of respiratory failure.
In a “foot drop” gait (choice A), the patients must lift their legs high enough to avoid tripping over their drooping feet. It is characteristic of peripheral neuropathy, which typically causes distal muscle weakness rather than proximal muscle weakness.
Deep tendon reflex (choice C) is usually not affected in muscle dystrophy.
Myotonia (choice D) is used to describe slow relaxation of the muscle after contraction and is characteristic of myotonic muscular dystrophy.
The Babinski sign (choice E), also called the extensor plantar reflex, is normal before 18 months of age, but signifies damage of the upper motor neurons in patients older than this. It is also associated with spasticity and increased reflexes.Incorrect
The correct answer is B. Duchenne muscular dystrophy is the most common type of muscular dystrophies. It is an X-linked genetic disorder and happens in 1:3600 boys. It is characterized by progressive muscle weakness caused by degeneration of muscle fibers. The disease is rarely symptomatic at birth, but usually develops to a clinically evident stage before the age of 5 years. Weakness is prominent in proximal muscles. Affected children often present with toe-walking and difficulty climbing stairs. The gait is waddling, and patients often fall. The Gower sign is very characteristic of Duchenne muscular dystrophy. It is considered positive if the patient uses his hands to “walk” up the legs when going from a prone to an upright sitting position because he does not have enough proximal muscle power to get up in a normal fashion. Pseudohypertrophy of the calves is also a prominent feature. Many patients will die in their 20s because of respiratory failure.
In a “foot drop” gait (choice A), the patients must lift their legs high enough to avoid tripping over their drooping feet. It is characteristic of peripheral neuropathy, which typically causes distal muscle weakness rather than proximal muscle weakness.
Deep tendon reflex (choice C) is usually not affected in muscle dystrophy.
Myotonia (choice D) is used to describe slow relaxation of the muscle after contraction and is characteristic of myotonic muscular dystrophy.
The Babinski sign (choice E), also called the extensor plantar reflex, is normal before 18 months of age, but signifies damage of the upper motor neurons in patients older than this. It is also associated with spasticity and increased reflexes. -
Question 50 of 151
50. Question
A 64-year-old man is undergoing chemotherapy and has occasional nausea and vomiting, which are treated with IV prochlorperazine. After several days of therapy, the patient complains that he feels very restless and agitated and he cannot stop moving his legs. Which of the following medications, if given during his initial regimen, might have best prevented this reaction?
Correct
The correct answer is C. This patient has evidence of akathisia, the feeling of restlessness that sometimes occurs as a reaction to use of neuroleptic medications, such as prochlorperazine and haloperidol. Prevention of akathisia is most reliably maintained by use of a benzodiazepine medication, such as lorazepam given IV during administration of the neuroleptic drug. This is particularly important in a patient whose immune system and metabolic capacity may show signs of compromise.
Chlorpromazine (choice A) and haloperidol (choice B) are both neuroleptic medications that would exacerbate akathisia rather than prevent it.
Paroxetine (choice D) and sertraline (choice E) are both antidepressant medications that are specific serotonergic reuptake inhibitors (SSRIs). These agents have no effect on the prevention of neuroleptic-induced akathisia.Incorrect
The correct answer is C. This patient has evidence of akathisia, the feeling of restlessness that sometimes occurs as a reaction to use of neuroleptic medications, such as prochlorperazine and haloperidol. Prevention of akathisia is most reliably maintained by use of a benzodiazepine medication, such as lorazepam given IV during administration of the neuroleptic drug. This is particularly important in a patient whose immune system and metabolic capacity may show signs of compromise.
Chlorpromazine (choice A) and haloperidol (choice B) are both neuroleptic medications that would exacerbate akathisia rather than prevent it.
Paroxetine (choice D) and sertraline (choice E) are both antidepressant medications that are specific serotonergic reuptake inhibitors (SSRIs). These agents have no effect on the prevention of neuroleptic-induced akathisia. -
Question 51 of 151
51. Question
A 38-year-old woman, gravida 1, para 0, at 8 weeks’ gestation comes to the physician for a prenatal visit. She has had no bleeding from the vagina or abdominal pain and no complaints. She has a long history of migraine headache and recently developed peptic ulcer disease (PUD). Examination shows a nontender 8-week sized uterus but is otherwise unremarkable. The patient is very concerned that her migraine headaches and peptic ulcer disease will make her pregnancy intolerable. Which of the following is the most appropriate response?
Correct
The correct answer is A. Within the last decade it has been recognized that Helicobacter pylori plays a central role in the pathogenesis of chronic gastritis and peptic ulcer disease (PUD). Acid secretion is also known to play a role. During pregnancy, gastric acid secretion is reduced and there is also a decrease in gastric motility. Pregnancy is also associated with increased mucus secretion, which is felt to have a protective effect on the gastrointestinal tract. Because of these physiologic changes during pregnancy, active peptic ulcer disease during pregnancy is extremely uncommon during pregnancy. Women rarely develop PUD in pregnancy and women with PUD note considerable improvement. Estimates are that 90% of patients with active PUD will experience remission during pregnancy. However, once the pregnancy is completed, almost all women will experience recurrence in the next few years.
A significant portion (up to 20%) of women experience migraine headaches during their lives, so issues regarding migraines and pregnancy are not uncommon. As with PUD, there is usually a dramatic improvement of migraines during pregnancy. Estimates are that 70% of women with migraines will have improvement. However, it is interesting to note that some women will have their first experience with migraine during pregnancy and may only experience migraine with pregnancy. Migraine headache during pregnancy should be treated with acetaminophen and antiemetics. Codeine or meperidine may be given for severe headaches. Ergotamine preparations should be avoided in pregnancy. The safety of sumatriptan during pregnancy has not been established, so pregnant patients, at present, should seek alternative medications.
To state that pregnancy is associated with worsening of migraines and PUD (choice B), worsening migraines and improved PUD (choice C), or improved migraines and worsened PUD (choice D) is incorrect. As explained above, pregnancy is associated with both improved migraines and PUD.
To state that pregnancy has no effect on migraines or PUD (choice E) is also incorrect. These two illnesses are examples of the profound effect that pregnancy can have on certain conditions.Incorrect
The correct answer is A. Within the last decade it has been recognized that Helicobacter pylori plays a central role in the pathogenesis of chronic gastritis and peptic ulcer disease (PUD). Acid secretion is also known to play a role. During pregnancy, gastric acid secretion is reduced and there is also a decrease in gastric motility. Pregnancy is also associated with increased mucus secretion, which is felt to have a protective effect on the gastrointestinal tract. Because of these physiologic changes during pregnancy, active peptic ulcer disease during pregnancy is extremely uncommon during pregnancy. Women rarely develop PUD in pregnancy and women with PUD note considerable improvement. Estimates are that 90% of patients with active PUD will experience remission during pregnancy. However, once the pregnancy is completed, almost all women will experience recurrence in the next few years.
A significant portion (up to 20%) of women experience migraine headaches during their lives, so issues regarding migraines and pregnancy are not uncommon. As with PUD, there is usually a dramatic improvement of migraines during pregnancy. Estimates are that 70% of women with migraines will have improvement. However, it is interesting to note that some women will have their first experience with migraine during pregnancy and may only experience migraine with pregnancy. Migraine headache during pregnancy should be treated with acetaminophen and antiemetics. Codeine or meperidine may be given for severe headaches. Ergotamine preparations should be avoided in pregnancy. The safety of sumatriptan during pregnancy has not been established, so pregnant patients, at present, should seek alternative medications.
To state that pregnancy is associated with worsening of migraines and PUD (choice B), worsening migraines and improved PUD (choice C), or improved migraines and worsened PUD (choice D) is incorrect. As explained above, pregnancy is associated with both improved migraines and PUD.
To state that pregnancy has no effect on migraines or PUD (choice E) is also incorrect. These two illnesses are examples of the profound effect that pregnancy can have on certain conditions. -
Question 52 of 151
52. Question
During the performance of a supraclavicular node biopsy under local anesthesia, a hissing sound is suddenly heard, and the patient suddenly dies. At the time of the catastrophic event, the target node was under traction, and the final cut was being made blindly behind it to free it up completely. The patient, an otherwise healthy 24-year-old man, was inhaling at that moment. Which of the following most likely caused this patient’s death?
Correct
The correct answer is B. Major veins at the base of the neck have negative pressure during inspiration and, if injured at that moment, will suck air rather than bleed. The air embolism then leads to sudden death.
Arterial injury (choice A) would have led to massive bleeding but not to sudden death.
Pneumothorax (choice C) can indeed happen when surgery is being done in the supraclavicular area, and a sucking sound might even be heard.
However, sudden lung collapse in a young, healthy person leads to dyspnea, not to sudden death. Sympathetic discharge (choice D) would be hard to produce while pulling and dissecting a node. If it were done, however, there would be vasoconstriction, tachycardia, perspiration, and hypertension, rather than sudden death.
Had the trachea been injured (choice E), essentially nothing would have happened at the time.Incorrect
The correct answer is B. Major veins at the base of the neck have negative pressure during inspiration and, if injured at that moment, will suck air rather than bleed. The air embolism then leads to sudden death.
Arterial injury (choice A) would have led to massive bleeding but not to sudden death.
Pneumothorax (choice C) can indeed happen when surgery is being done in the supraclavicular area, and a sucking sound might even be heard.
However, sudden lung collapse in a young, healthy person leads to dyspnea, not to sudden death. Sympathetic discharge (choice D) would be hard to produce while pulling and dissecting a node. If it were done, however, there would be vasoconstriction, tachycardia, perspiration, and hypertension, rather than sudden death.
Had the trachea been injured (choice E), essentially nothing would have happened at the time. -
Question 53 of 151
53. Question
57-year-old woman presents with progressive shortness of breath over the past 2 days. The woman was admitted to the medical service 6 days ago after a fall and has been on bed rest for a nondisplaced pubic ramus fracture. She has been on deep vein thrombosis prophylaxis with subcutaneous heparin. Her past medical history is significant for type 2 diabetes and dialysis-dependent renal failure secondary to diabetic nephropathy. She makes no urine at baseline. Her last dialysis run was 4 days ago, though she usually undergoes dialysis 3 times per week. She has no chest pain. On physical examination, she appears anxious. Her blood pressure is 160/105 mm Hg, pulse is 110/min, and respirations are 22/min. Her oxygen saturation is 80% on room air, and she appears cyanotic. She has a jugular venous pressure of 10 cm and inspiratory crackles half way up from the bases on auscultation of the lungs. An ECG reveals a rate-related right bundle branch block but no ischemic changes. A chest x-ray film obtained yesterday revealed interstitial edema and vascular redistribution to the apices. Which of the following is the most appropriate initial therapy?
Correct
The correct answer is E. Although the patient clearly has abnormal vital signs that are worrisome, the finding most likely to place the patient in immediate danger is her hypoxia. The patient’s oxygen saturation of 80% places her at significant risk for delirium, cardiac arrhythmias, and cardiopulmonary arrest. Oxygen should first be administered noninvasively in this case, starting with a non-rebreather face mask until the clinical picture can be stabilized.
An anti-hypertensive agent (choice A) would be an important therapy if her congestive heart failure and hypoxia were the result of diastolic ventricular dysfunction from a hypertensive emergency. Even if this were the case, however, the initial therapy would still be to initially treat her hypoxia while administering an anti-hypertensive agent.
Beta blockade (choice B) is effective at controlling heart rate through the drug’s actions on nodal conduction in the heart. However, there is rarely an indication to treat sinus tachycardia, as the underlying cause (in this case likely anxiety and the increased work of breathing against non-compliant lungs) should be addressed first. Again, the hypoxia takes precedence.
IV morphine (choice C) would have the effect here of diminishing anxiety and decreasing venous return to the heart. Although these effects may be attractive, morphine is also a respiratory depressant, and the patient may require her complete respiratory drive to maintain her blood oxygen levels.
Of all of the ways to administer oxygen, intubation, either endotracheal (choice D) or nasotracheal, is the only means to ensure 100% oxygen delivery to the lungs. This patient may ultimately require intubation, but, given its risks, it is appropriate to attempt noninvasive oxygen delivery first. If the question had suggested that the patient was unable to maintain her airway or that noninvasive methods had failed to correct the hypoxia, this choice would have been correct.Incorrect
The correct answer is E. Although the patient clearly has abnormal vital signs that are worrisome, the finding most likely to place the patient in immediate danger is her hypoxia. The patient’s oxygen saturation of 80% places her at significant risk for delirium, cardiac arrhythmias, and cardiopulmonary arrest. Oxygen should first be administered noninvasively in this case, starting with a non-rebreather face mask until the clinical picture can be stabilized.
An anti-hypertensive agent (choice A) would be an important therapy if her congestive heart failure and hypoxia were the result of diastolic ventricular dysfunction from a hypertensive emergency. Even if this were the case, however, the initial therapy would still be to initially treat her hypoxia while administering an anti-hypertensive agent.
Beta blockade (choice B) is effective at controlling heart rate through the drug’s actions on nodal conduction in the heart. However, there is rarely an indication to treat sinus tachycardia, as the underlying cause (in this case likely anxiety and the increased work of breathing against non-compliant lungs) should be addressed first. Again, the hypoxia takes precedence.
IV morphine (choice C) would have the effect here of diminishing anxiety and decreasing venous return to the heart. Although these effects may be attractive, morphine is also a respiratory depressant, and the patient may require her complete respiratory drive to maintain her blood oxygen levels.
Of all of the ways to administer oxygen, intubation, either endotracheal (choice D) or nasotracheal, is the only means to ensure 100% oxygen delivery to the lungs. This patient may ultimately require intubation, but, given its risks, it is appropriate to attempt noninvasive oxygen delivery first. If the question had suggested that the patient was unable to maintain her airway or that noninvasive methods had failed to correct the hypoxia, this choice would have been correct. -
Question 54 of 151
54. Question
A 60-year-old man presents to the emergency room with severe abdominal pain. Physical examination demonstrates tender, smooth hepatomegaly and mild jaundice. Serum chemistries demonstrate an aspartate aminotransferase (AST) and alanine aminotransferase (ALT) of over ten times the upper limit of normal. The hematocrit is 63%. The patient has no known history of smoking or lung disease. Complete blood count shows an increased number in all cell lines, with the most marked elevation in the erythrocyte line. The morphology of the cells in the peripheral smear is normal. Which of the following is the most likely cause of this patient’s liver function abnormalities?
Correct
The correct answer is B. This patient has probable polycythemia vera (a below normal erythropoietin level would be a helpful confirmatory study) which is presenting with Budd-Chiari syndrome. Budd-Chiari syndrome is a rare disorder with high mortality rate usually caused by the thrombosis of major hepatic veins. It can present either acutely, as in this case, or more slowly with chronic vague abdominal pain and portal hypertension. Underlying predisposing causes include myeloproliferative disorders (notably polycythemia vera, as in this case); other hematologic disorders (sickle cell disease, paroxysmal nocturnal hemoglobinuria); defective inhibition of coagulation (antiphospholipid antibodies, low antithrombin III, low protein C, low protein S, and low factor V Leiden); possibly estrogens (oral contraceptives, pregnancy); and local liver disease (malignancies, trauma, or suppurative lesions).
Alcoholic cirrhosis (choice A) can cause jaundice, but AST and ALT elevations are usually not very high. Further, nothing in this man’s history suggests significant alcohol use.
Cholelithiasis (choice C) can cause obstructive jaundice when a small stone occludes a bile duct, but AST and ALT elevations are usually not very high.
No risk factors are noted for either chronic hepatits B (choice D) or fulminant hepatitis A (choice E). Chronic viral hepatitis does not usually have very high elevations of AST or ALT, but fulminant hepatitis, which Budd-Chiari syndrome mimics, can.Incorrect
The correct answer is B. This patient has probable polycythemia vera (a below normal erythropoietin level would be a helpful confirmatory study) which is presenting with Budd-Chiari syndrome. Budd-Chiari syndrome is a rare disorder with high mortality rate usually caused by the thrombosis of major hepatic veins. It can present either acutely, as in this case, or more slowly with chronic vague abdominal pain and portal hypertension. Underlying predisposing causes include myeloproliferative disorders (notably polycythemia vera, as in this case); other hematologic disorders (sickle cell disease, paroxysmal nocturnal hemoglobinuria); defective inhibition of coagulation (antiphospholipid antibodies, low antithrombin III, low protein C, low protein S, and low factor V Leiden); possibly estrogens (oral contraceptives, pregnancy); and local liver disease (malignancies, trauma, or suppurative lesions).
Alcoholic cirrhosis (choice A) can cause jaundice, but AST and ALT elevations are usually not very high. Further, nothing in this man’s history suggests significant alcohol use.
Cholelithiasis (choice C) can cause obstructive jaundice when a small stone occludes a bile duct, but AST and ALT elevations are usually not very high.
No risk factors are noted for either chronic hepatits B (choice D) or fulminant hepatitis A (choice E). Chronic viral hepatitis does not usually have very high elevations of AST or ALT, but fulminant hepatitis, which Budd-Chiari syndrome mimics, can. -
Question 55 of 151
55. Question
60-year-old woman consults a physician because of weakness, headaches, dizziness, and tingling in her hands and feet. Physical examination demonstrates multiple areas of bruising on the back of her forearms and shins. On specific questioning, she reports having had five nosebleeds in the past two months, which she had attributed to “dry air”. Blood studies are drawn which show a platelet count of 1.2 ¢² 106/?L, a red cell count of 5.1 ¢²106/?L, and a white count of 10,500/?L with a normal differential count. Review of the peripheral smear demonstrates many abnormally large platelets, platelet aggregates, and megakaryocyte fragments. No abnormal red or white blood cells are seen. Philadelphia chromosome studies are negative. Which of the following is the most likely diagnosis?
Correct
The correct answer is D. The most likely diagnosis is primary (essential) thrombocythemia. The condition is due to a clonal abnormality of a multipotent hematopoietic cell that produces megakaryocytic hyperplasia with resultant increased platelet count. Since the platelets are often abnormal, either a thrombotic or a hemorrhagic tendency may be seen. The platelet count may be as low as 500,000/?L or greater than 1,000,000/?L. The clinical presentation and laboratory findings illustrated in the question stem are typical. The other choices listed commonly must be excluded before a diagnosis of primary thrombocythemia is confirmed. Chronic myelogenous leukemia (choice A) can be a cause of increased platelet count, but the absence of either a Philadelphia chromosome or a markedly increased white count argues against this possibility.
Myelofibrosis (choice B) can also cause thrombocythemia, but would likely show some abnormally shaped (often tear drops) red cells. Polycythemia vera (choice C) can also cause thrombocythemia, but would be associated with an increased red cell mass.
Secondary thrombocythemia (choice E) is a reactive process that may occur in a variety of settings including chronic inflammatory disorders, acute infection, hemorrhage or hemolysis, tumors, iron deficiency, or splenectomy. Abnormal platelet forms are not usually seen on smears from these patients and platelet function tests are usually normal.Incorrect
The correct answer is D. The most likely diagnosis is primary (essential) thrombocythemia. The condition is due to a clonal abnormality of a multipotent hematopoietic cell that produces megakaryocytic hyperplasia with resultant increased platelet count. Since the platelets are often abnormal, either a thrombotic or a hemorrhagic tendency may be seen. The platelet count may be as low as 500,000/?L or greater than 1,000,000/?L. The clinical presentation and laboratory findings illustrated in the question stem are typical. The other choices listed commonly must be excluded before a diagnosis of primary thrombocythemia is confirmed. Chronic myelogenous leukemia (choice A) can be a cause of increased platelet count, but the absence of either a Philadelphia chromosome or a markedly increased white count argues against this possibility.
Myelofibrosis (choice B) can also cause thrombocythemia, but would likely show some abnormally shaped (often tear drops) red cells. Polycythemia vera (choice C) can also cause thrombocythemia, but would be associated with an increased red cell mass.
Secondary thrombocythemia (choice E) is a reactive process that may occur in a variety of settings including chronic inflammatory disorders, acute infection, hemorrhage or hemolysis, tumors, iron deficiency, or splenectomy. Abnormal platelet forms are not usually seen on smears from these patients and platelet function tests are usually normal. -
Question 56 of 151
56. Question
A 31-year-old male immigrant from India is found on a routine physical examination to have a single, 2-cm nodule in the right lobe of his thyroid gland. The mass is firm, moves up and down with swallowing, and is not tender. The skin of his face and neck is pitted with multiple scars, which suggest smallpox; however, he explains that the scars are due to very severe acne that he had as a youngster, for which he eventually received external beam radiation therapy at the age of 14. His thyroid function tests are normal, and a fine needle aspiration (FNA) cytology of the mass is read by the pathologist as “indeterminate.” Which of the following is the most appropriate next step in management?
Correct
The correct answer is E. The patient is at high risk for thyroid cancer (young, male, with a single nodule and a history of radiation), and a reading of “indeterminate” in an FNA is a surgical indication.
No further care (choice A) is totally wrong. It assumes that normal thyroid function means there is nothing wrong with the thyroid, when in fact thyroid cancer almost never alters thyroid function. This choice also assumes that if an FNA is not read as cancer, the patient does not have that disease.
Focusing on function (choice B) as the criterion to do something is wrong for the same reasons.
Thyroid scan and sonogram (choice C) were formerly valuable criteria to select surgical candidates (cold solid nodules meant a high risk of cancer), but the FNA provides a higher yield of malignancy in resected specimens, and thus has rendered the other tests obsolete for this purpose.
Repeating the FNA (choice D) assumes that, given more cells, the pathologist should be able to distinguish benign from malignant. The pathologist has no trouble recognizing malignant features in papillary, medullary, or anaplastic cancers of the thyroid, but cannot do so with follicular neoplasms. Follicular adenoma and follicular carcinoma require a look at the entire specimen to tell them apartIncorrect
The correct answer is E. The patient is at high risk for thyroid cancer (young, male, with a single nodule and a history of radiation), and a reading of “indeterminate” in an FNA is a surgical indication.
No further care (choice A) is totally wrong. It assumes that normal thyroid function means there is nothing wrong with the thyroid, when in fact thyroid cancer almost never alters thyroid function. This choice also assumes that if an FNA is not read as cancer, the patient does not have that disease.
Focusing on function (choice B) as the criterion to do something is wrong for the same reasons.
Thyroid scan and sonogram (choice C) were formerly valuable criteria to select surgical candidates (cold solid nodules meant a high risk of cancer), but the FNA provides a higher yield of malignancy in resected specimens, and thus has rendered the other tests obsolete for this purpose.
Repeating the FNA (choice D) assumes that, given more cells, the pathologist should be able to distinguish benign from malignant. The pathologist has no trouble recognizing malignant features in papillary, medullary, or anaplastic cancers of the thyroid, but cannot do so with follicular neoplasms. Follicular adenoma and follicular carcinoma require a look at the entire specimen to tell them apart -
Question 57 of 151
57. Question
A 25-year-old woman with a several-year history of binging and purging presents to a psychiatrist complaining of a lack of energy, poor sleep, and decreased ability to concentrate. She is very concerned about weight gain. Which of the following medications would be the most appropriate to initiate?
Correct
The correct answer is B. This patient appears to have symptoms of major depression in the context of bulimia. She should therefore be treated with an antidepressant medication, and a selective serotonin reuptake inhibitor (SSRI), such as fluoxetine, would be most appropriate.
Bupropion (choice A) is an antidepressant, but it is contraindicated in patients with binging and purging behavior because of its lowering of the seizure threshold in patients with eating disorders.
Haloperidol (choice C) is an antipsychotic medication and is not indicated in major depression. Lithium carbonate (choice D) is sometimes used to treat depression, but only in the context of bipolar disorder. Its usual indication is for the treatment of mania.
Valproic acid (choice E) is an anticonvulsant used as a mood stabilizer in the treatment of bipolar disorder.Incorrect
The correct answer is B. This patient appears to have symptoms of major depression in the context of bulimia. She should therefore be treated with an antidepressant medication, and a selective serotonin reuptake inhibitor (SSRI), such as fluoxetine, would be most appropriate.
Bupropion (choice A) is an antidepressant, but it is contraindicated in patients with binging and purging behavior because of its lowering of the seizure threshold in patients with eating disorders.
Haloperidol (choice C) is an antipsychotic medication and is not indicated in major depression. Lithium carbonate (choice D) is sometimes used to treat depression, but only in the context of bipolar disorder. Its usual indication is for the treatment of mania.
Valproic acid (choice E) is an anticonvulsant used as a mood stabilizer in the treatment of bipolar disorder. -
Question 58 of 151
58. Question
A 15-year-old boy comes to the physician for advice about his facial acne. On examination, the patient has mild to moderate acne, mostly consisting of open comedones, some closed comedones, and a few pustules on the forehead and cheeks. Which of the following is the best advice to give this patient?
Correct
The correct answer is C. Acne vulgaris affects the majority of adolescents and is more prevalent in males. Hormonal influences, abnormal keratinization of pilosebaceous units and colonization by bacteria (Propionibacterium acnes) are important pathogenetic elements. Treatment of acne depends on the severity of the condition. Topical application of comedolytic agents such as retinoids (tretinoin, adapalene, and the new yeast-derived agent azaleic acid) is effective for mild to moderate forms of non-inflammatory acne, characterized by open comedones. Daily application of these compounds will result in improvement within several weeks after starting treatment. Mild skin irritation and scaling may be minimized by starting with low-concentration creams, and then progressively increasing the concentration. Another side effect of retinoids is increased photosensitivity. The patient must be instructed to avoid prolonged exposure to the sun and to use a sunscreen.
Avoidance of chocolate and spicy foods (choice A) would have no beneficial effects on acne. It is well established that there is no correlation between acne and specific types of foods.
Frequent face washing with strong soap (choice B) will probably cause exacerbation of acne. This skin condition is not caused by dirt. Gentle face washing once or twice daily with mild soaps is recommended.
Treatment with oral antibiotic (choice D) is aimed at decreasing bacterial colonization It is used for patients who fail to respond to topical treatments or have severe forms of inflammatory acne. The antibiotics of choice include tetracyclines and erythromycin.
Oral treatment with isotretinoin (choice E) is used for severe cases of acne not responding to topical comedolytics and antibiotics. This compound acts by decreasing sebum production. In addition to various types of side effects, isotretinoin is teratogenic. Female patients of childbearing age should be required to use effective means of contraception beginning one month before treatment to one month after treatment.Incorrect
The correct answer is C. Acne vulgaris affects the majority of adolescents and is more prevalent in males. Hormonal influences, abnormal keratinization of pilosebaceous units and colonization by bacteria (Propionibacterium acnes) are important pathogenetic elements. Treatment of acne depends on the severity of the condition. Topical application of comedolytic agents such as retinoids (tretinoin, adapalene, and the new yeast-derived agent azaleic acid) is effective for mild to moderate forms of non-inflammatory acne, characterized by open comedones. Daily application of these compounds will result in improvement within several weeks after starting treatment. Mild skin irritation and scaling may be minimized by starting with low-concentration creams, and then progressively increasing the concentration. Another side effect of retinoids is increased photosensitivity. The patient must be instructed to avoid prolonged exposure to the sun and to use a sunscreen.
Avoidance of chocolate and spicy foods (choice A) would have no beneficial effects on acne. It is well established that there is no correlation between acne and specific types of foods.
Frequent face washing with strong soap (choice B) will probably cause exacerbation of acne. This skin condition is not caused by dirt. Gentle face washing once or twice daily with mild soaps is recommended.
Treatment with oral antibiotic (choice D) is aimed at decreasing bacterial colonization It is used for patients who fail to respond to topical treatments or have severe forms of inflammatory acne. The antibiotics of choice include tetracyclines and erythromycin.
Oral treatment with isotretinoin (choice E) is used for severe cases of acne not responding to topical comedolytics and antibiotics. This compound acts by decreasing sebum production. In addition to various types of side effects, isotretinoin is teratogenic. Female patients of childbearing age should be required to use effective means of contraception beginning one month before treatment to one month after treatment. -
Question 59 of 151
59. Question
In preparation for an inguinal hernia repair, a 22-year-old man has a spinal anesthetic placed. The level of sensory block turns out to be much higher than had been planned, and shortly thereafter his blood pressure drops to 75/20 mm Hg. He looks warm and flushed, and his central venous pressure is near zero. Which of the following should be included in his therapy?
Correct
The correct answer is D. A high spinal anesthetic can produce vasomotor shock by inducing widespread vasodilation. Vasoconstrictors are the appropriate therapy, but since the capacity of the vascular tree is also increased under these circumstances, filling it up with additional volume is also helpful.
Diuretics and fluid restriction (choice A) would compound the problem. The patient needs more fluid, not less.
Whole blood and clotting factors (choice B) are not needed. The volume can be increased with cheaper and safer IV fluids until proper vascular tone is restored. Coagulation factors have not been lost.
Inotropic agents and mechanical assistance to the circulation (choice C) are indicated in cardiogenic shock, which would be rare in a 22-year-old and would be identified, among other things, by a high central venous pressure.
Vasodilators and fluids (choice E) are sometimes a good combination when hypovolemia plus high peripheral resistance deprive tissues of proper perfusion. In this case, however, vasodilation already exists (and is the genesis of the problem).Incorrect
The correct answer is D. A high spinal anesthetic can produce vasomotor shock by inducing widespread vasodilation. Vasoconstrictors are the appropriate therapy, but since the capacity of the vascular tree is also increased under these circumstances, filling it up with additional volume is also helpful.
Diuretics and fluid restriction (choice A) would compound the problem. The patient needs more fluid, not less.
Whole blood and clotting factors (choice B) are not needed. The volume can be increased with cheaper and safer IV fluids until proper vascular tone is restored. Coagulation factors have not been lost.
Inotropic agents and mechanical assistance to the circulation (choice C) are indicated in cardiogenic shock, which would be rare in a 22-year-old and would be identified, among other things, by a high central venous pressure.
Vasodilators and fluids (choice E) are sometimes a good combination when hypovolemia plus high peripheral resistance deprive tissues of proper perfusion. In this case, however, vasodilation already exists (and is the genesis of the problem). -
Question 60 of 151
60. Question
A 42-year-old man is diagnosed by his primary care physician as having major depressive disorder. The patient tells his physician that he is extremely concerned about his sexual performance, as he is worried that he is getting older and that he is having some marital difficulties with his wife. Which of the following antidepressants would be the best choice for initial treatment?
Correct
The correct answer is A. Bupropion is the antidepressant of the choices listed that has the fewest adverse sexual side effects.
Selective serotonin reuptake inhibitors, such as fluoxetine (choice B), paroxetine (choice D), and sertraline (choice E) are all known to be associated with erectile and orgasmic disturbances, such as delayed ejaculation and anorgasmia.
Because imipramine (choice C) also has effect on the serotonin levels in the brain, it too has sexual side effects, although to a somewhat lesser degree than the SSRIs.Incorrect
The correct answer is A. Bupropion is the antidepressant of the choices listed that has the fewest adverse sexual side effects.
Selective serotonin reuptake inhibitors, such as fluoxetine (choice B), paroxetine (choice D), and sertraline (choice E) are all known to be associated with erectile and orgasmic disturbances, such as delayed ejaculation and anorgasmia.
Because imipramine (choice C) also has effect on the serotonin levels in the brain, it too has sexual side effects, although to a somewhat lesser degree than the SSRIs. -
Question 61 of 151
61. Question
A 71-year-old woman is being treated for a severe chronic obstructive pulmonary disease (COPD) flare. The patient presented to the hospital 3 days ago with cough, fever, and pleuritic chest pain. She had been feeling fatigued and, on the day of admission, was persistently febrile. She has had more severe shortness of breath, often at rest and with mild exertion, and moderate dyspnea. A chest radiograph revealed a left lower lobe infiltrate, and she was started on antibiotics. The patient has a long smoking history with a forced expiration in 1 second (FEV1) of 1.1 L. Which of the following therapies is most beneficial with respect to long-term morbidity and mortality in this patient?
Correct
The correct answer is C. Two cardinal studies have shown a clear mortality benefit for hypoxic patients with chronic obstructive pulmonary disease (COPD) when they use long-term oxygen therapy (LTOT). These studies have shown that LTOT can increase body weight, alleviate cor pulmonale, decrease polycythemia, strengthen cardiac function, and improve exercise tolerance. Beta agonists (choice A) are widely prescribed as inhalers on an as-needed basis, but there is no evidence that this therapy is beneficial in any regard other than symptom relief.
Although there has been moderate debate about the use of systemic steroids, the use of inhaled corticosteroids (choice B) is fraught with even more issues and disagreement. Needless to say, even though these agents are used in the treatment of COPD patients, no data exist as to their long-term benefits or harm. There has been a long-standing debate about the utility of systemic corticosteroids (choice D). It is now clear, however, that these agents have a beneficial effect in shortening the duration of COPD flares, but not in the therapy of stable disease.
Theophylline (choice E) has been used in the treatment of reversible airway obstruction (asthma) and COPD for a number of years. Recently, it has again become a useful agent in the therapy of COPD, in both stable and flare attacks. Its effect on long-term morbidity and mortality has yet to be documented, however.Incorrect
The correct answer is C. Two cardinal studies have shown a clear mortality benefit for hypoxic patients with chronic obstructive pulmonary disease (COPD) when they use long-term oxygen therapy (LTOT). These studies have shown that LTOT can increase body weight, alleviate cor pulmonale, decrease polycythemia, strengthen cardiac function, and improve exercise tolerance. Beta agonists (choice A) are widely prescribed as inhalers on an as-needed basis, but there is no evidence that this therapy is beneficial in any regard other than symptom relief.
Although there has been moderate debate about the use of systemic steroids, the use of inhaled corticosteroids (choice B) is fraught with even more issues and disagreement. Needless to say, even though these agents are used in the treatment of COPD patients, no data exist as to their long-term benefits or harm. There has been a long-standing debate about the utility of systemic corticosteroids (choice D). It is now clear, however, that these agents have a beneficial effect in shortening the duration of COPD flares, but not in the therapy of stable disease.
Theophylline (choice E) has been used in the treatment of reversible airway obstruction (asthma) and COPD for a number of years. Recently, it has again become a useful agent in the therapy of COPD, in both stable and flare attacks. Its effect on long-term morbidity and mortality has yet to be documented, however. -
Question 62 of 151
62. Question
A 70-year-old woman with aortic sclerosis is admitted with chest pain. An infarct is ruled out by cardiac enzymes, but the patient has recurrent symptoms when weaned off heparin. On hospitalization day 2, she has right arm pain, absent brachial pulse on the right, and a cold distal right arm. Her hematocrit is 34%, and platelets are 30,000/mm3. Her partial thromboplastin time is 64 sec. Which of the following is the most likely cause of this patient’s absent brachial pulse?
Correct
The correct answer is B. Heparin-induced thrombocytopenia (HIT) is the result of platelet aggregation caused by heparin-induced antibodies. It is seen in 1% to 5% of patients on heparin. Therapy is discontinuation of the heparin and use of another anticoagulant, such as lepirudin. When the platelet count falls below 50,000/mm3 the heparin should be stopped. HIT can lead to limb-threatening thromboses, as in this patient, and constitutes a medical emergency. Arterial thrombosis is a manifestation of the HIT syndrome.
Septic emboli or marantic emboli (choice A) may cause an ischemic limb as a result of occlusion of an artery. Treatment involves treatment of the underlying condition; valve replacement may be needed. Aortic sclerosis is the result of calcification or sclerosis of the aortic valve and is not associated with embolic events. Paradoxical emboli (choice C) may be the result of a deep venous thrombosis passing through a patent foramen ovale and may be a cause of arterial occlusion as the clot moves from the venous to the arterial system. Treatment requires closure of the patent foramen ovale, either surgically or through catheterization.
Hypercoagulability from immobilization (choice D) typically causes deep venous thromboses. In such a situation, placement of a vena cava filter must be considered. Conditions predisposing a patient to hypercoagulability include trauma, stasis, and conditions such as cancer.
Brachial artery vasospasm (choice E) is a rare occurrence and should be transient. The patient may complain of hand discomfort, but, given the transient nature of the condition, a pulse should still be palpated and the extremity should not be cool. A patient with Raynaud’s phenomenon may exhibit such symptoms.Incorrect
The correct answer is B. Heparin-induced thrombocytopenia (HIT) is the result of platelet aggregation caused by heparin-induced antibodies. It is seen in 1% to 5% of patients on heparin. Therapy is discontinuation of the heparin and use of another anticoagulant, such as lepirudin. When the platelet count falls below 50,000/mm3 the heparin should be stopped. HIT can lead to limb-threatening thromboses, as in this patient, and constitutes a medical emergency. Arterial thrombosis is a manifestation of the HIT syndrome.
Septic emboli or marantic emboli (choice A) may cause an ischemic limb as a result of occlusion of an artery. Treatment involves treatment of the underlying condition; valve replacement may be needed. Aortic sclerosis is the result of calcification or sclerosis of the aortic valve and is not associated with embolic events. Paradoxical emboli (choice C) may be the result of a deep venous thrombosis passing through a patent foramen ovale and may be a cause of arterial occlusion as the clot moves from the venous to the arterial system. Treatment requires closure of the patent foramen ovale, either surgically or through catheterization.
Hypercoagulability from immobilization (choice D) typically causes deep venous thromboses. In such a situation, placement of a vena cava filter must be considered. Conditions predisposing a patient to hypercoagulability include trauma, stasis, and conditions such as cancer.
Brachial artery vasospasm (choice E) is a rare occurrence and should be transient. The patient may complain of hand discomfort, but, given the transient nature of the condition, a pulse should still be palpated and the extremity should not be cool. A patient with Raynaud’s phenomenon may exhibit such symptoms. -
Question 63 of 151
63. Question
A 6-year-old boy is brought to the pediatrician because of a 3-day history of skin lesions. On physical examination, he has multiple yellow, crusted erosions below the nares and on the cheeks, chin, and upper extremities. The rest of the examination is normal. Which of the following is the most appropriate treatment for this condition?
Correct
The correct answer is C. Bullous impetigo (staphylococcal impetigo) is caused by an epidermolytic toxin produced at the site of infection, most commonly by staphylococci of phage group II. The toxin causes intraepidermal cleavage below or within the stratum granulosum. Bullous impetigo is most common in infants and children. It typically occurs on the face, but it may infect any body surface. There may be a few lesions localized in one area, or the lesions may be numerous and widely scattered. One or more vesicles enlarge rapidly to form bullae in which the contents turn from clear to cloudy. The center of the thin-roofed bulla collapses, and a thin, flat, honey-colored crust may appear in the center with a bright red, inflamed, moist base that oozes serum. In most cases, a tinea-like scaling border replaces the fluid-filled rim as the round lesions enlarge and become contiguous with the others .The border dries and forms a crust. The lesions have little or no surrounding erythema. Regional lymphadenitis is uncommon with pure staphylococcal impetigo. There is some evidence that the responsible staphylococci colonize the nose and then spread to normal skin prior to infection. Serious secondary infections (e.g., osteomyelitis, septic arthritis, and pneumonia) may follow seemingly innocuous superficial infections in infants. The drug of choice for impetigo is oral cephalexin. Cloxacillin, dicloxacillin, and azithromycin are good alteratives.
Because some cases of impetigo are due to a mixed staphylococcal/streptococcal infection penicillin and amoxicillin (choice B) are inadequate for treatment.
Oral acyclovir (choice A) is used to treat herpes simplex virus infection.
Topical ketoconazole (choice D) is used to treat fungal infection of the skin. Topical 2% hydrocortisone (choice E) is ineffective against impetigo.Incorrect
The correct answer is C. Bullous impetigo (staphylococcal impetigo) is caused by an epidermolytic toxin produced at the site of infection, most commonly by staphylococci of phage group II. The toxin causes intraepidermal cleavage below or within the stratum granulosum. Bullous impetigo is most common in infants and children. It typically occurs on the face, but it may infect any body surface. There may be a few lesions localized in one area, or the lesions may be numerous and widely scattered. One or more vesicles enlarge rapidly to form bullae in which the contents turn from clear to cloudy. The center of the thin-roofed bulla collapses, and a thin, flat, honey-colored crust may appear in the center with a bright red, inflamed, moist base that oozes serum. In most cases, a tinea-like scaling border replaces the fluid-filled rim as the round lesions enlarge and become contiguous with the others .The border dries and forms a crust. The lesions have little or no surrounding erythema. Regional lymphadenitis is uncommon with pure staphylococcal impetigo. There is some evidence that the responsible staphylococci colonize the nose and then spread to normal skin prior to infection. Serious secondary infections (e.g., osteomyelitis, septic arthritis, and pneumonia) may follow seemingly innocuous superficial infections in infants. The drug of choice for impetigo is oral cephalexin. Cloxacillin, dicloxacillin, and azithromycin are good alteratives.
Because some cases of impetigo are due to a mixed staphylococcal/streptococcal infection penicillin and amoxicillin (choice B) are inadequate for treatment.
Oral acyclovir (choice A) is used to treat herpes simplex virus infection.
Topical ketoconazole (choice D) is used to treat fungal infection of the skin. Topical 2% hydrocortisone (choice E) is ineffective against impetigo. -
Question 64 of 151
64. Question
19-year-old student is admitted to the psychiatry inpatient unit with the working diagnosis of first-break psychosis. Because of combativeness and threats to others, he is put in seclusion. At first, he refused to take medication by mouth; however, after attacking a nurse he is given haloperidol intramuscularly on two occasions. He has now developed acute torticollis and twitching of the mouth and face on that side. The family is furious, stating that the treatment caused the seizures. Which of the following reactions did this patient most likely have?
Correct
The correct answer is A. Acute dystonia is an involuntary spasm of a particular group of muscles that can involve the neck, jaw, tongue, eyes, or the entire body. It can be an early adverse effect of antipsychotics, and it is more common in younger men. It is more common with typical antipsychotics. The treatment of choice is parenteral administration of anticholinergics.
Akathisia (choice B) is a subjective feeling of muscle discomfort and restlessness that can cause agitation, pacing, anxiety, and dysphoria. It is related to the use of antipsychotics and can appear any time during treatment. It is treated by antipsychotic dose reduction, propranolol, or benzodiazepines.
Asterixis (choice C) consists of coarse arrhythmic lapses of sustained posture. It is related to metabolic disorders and is most easily seen when the patient’s arms are outstretched. Asterixis occurs bilaterally.
Lennox-Gastaut syndrome (choice D) is a variant of petit mal epilepsy and consists of intellectual impairment, distinctive slow spike and wave pattern, and atonic postural lapses followed by minor tonic-clonic spasms.
Pseudoseizures (choice E) are part of conversion disorder and are not directly related to the use of antipsychotics. They are very much like real seizures except that there is no aura and no EEG abnormalities. The movements are asynchronous and non-stereotyped, and they occur when the person is awake.Incorrect
The correct answer is A. Acute dystonia is an involuntary spasm of a particular group of muscles that can involve the neck, jaw, tongue, eyes, or the entire body. It can be an early adverse effect of antipsychotics, and it is more common in younger men. It is more common with typical antipsychotics. The treatment of choice is parenteral administration of anticholinergics.
Akathisia (choice B) is a subjective feeling of muscle discomfort and restlessness that can cause agitation, pacing, anxiety, and dysphoria. It is related to the use of antipsychotics and can appear any time during treatment. It is treated by antipsychotic dose reduction, propranolol, or benzodiazepines.
Asterixis (choice C) consists of coarse arrhythmic lapses of sustained posture. It is related to metabolic disorders and is most easily seen when the patient’s arms are outstretched. Asterixis occurs bilaterally.
Lennox-Gastaut syndrome (choice D) is a variant of petit mal epilepsy and consists of intellectual impairment, distinctive slow spike and wave pattern, and atonic postural lapses followed by minor tonic-clonic spasms.
Pseudoseizures (choice E) are part of conversion disorder and are not directly related to the use of antipsychotics. They are very much like real seizures except that there is no aura and no EEG abnormalities. The movements are asynchronous and non-stereotyped, and they occur when the person is awake. -
Question 65 of 151
65. Question
31-year-old, HIV-positive woman, gravida 3, para 2, at 32-weeks’ gestation comes to the physician for a prenatal visit. Her prenatal course is significant for the fact that she has taken zidovudine throughout the pregnancy. Otherwise, her prenatal course has been unremarkable. She has no history of mental illness. She states that she has been weighing the benefits and risks of cesarean delivery in preventing transmission of the virus to her baby. After much deliberation, she has decided that she does not want a cesarean delivery and would like to attempt a vaginal delivery. Which of the following is the most appropriate next step in management?
Correct
The correct answer is E. Cesarean delivery has been shown to decrease the rate of transmission of HIV from an infected mother to her fetus. Some reports have shown that the transmission rate can be decreased to as low as 2% with the combination of antiretroviral medication and elective cesarean delivery prior to labor or rupture of membranes. However, although cesarean delivery benefits the infant by decreasing the risk of transmission, the risks of the surgery accrue to the mother. Risks of cesarean delivery include bleeding, infection, and injury to internal organs. HIV-infected women with low CD4 cell counts are known to have higher rates of postoperative complications. Thus, the decision of whether to have a cesarean ultimately belongs to the mother. This patient’s autonomy must be respected and she should have a vaginal delivery if she so chooses.
To contact either psychiatry to evaluate the patient (choice A) or the hospital lawyers to get a court order for cesarean delivery (choice B) would be incorrect. Patient autonomy must be respected when it comes to the decision of whether to have a cesarean delivery. This patient has weighed the benefits and risks and has put a great deal of deliberation into her decision. There is no need to involve the psychiatry department or the hospital lawyers in this decision.
To perform cesarean delivery at 38 weeks (choice C) or once the patient is in labor (choice D) would not be correct. In HIV-positive women who do want an elective cesarean delivery, the delivery should be performed at 38 weeks to avoid the risk of labor or rupture of membranes. Once labor starts or the membranes are ruptured, the risk of HIV transmission increases. However, this patient does not want a cesarean delivery, so that operation should not be performed.Incorrect
The correct answer is E. Cesarean delivery has been shown to decrease the rate of transmission of HIV from an infected mother to her fetus. Some reports have shown that the transmission rate can be decreased to as low as 2% with the combination of antiretroviral medication and elective cesarean delivery prior to labor or rupture of membranes. However, although cesarean delivery benefits the infant by decreasing the risk of transmission, the risks of the surgery accrue to the mother. Risks of cesarean delivery include bleeding, infection, and injury to internal organs. HIV-infected women with low CD4 cell counts are known to have higher rates of postoperative complications. Thus, the decision of whether to have a cesarean ultimately belongs to the mother. This patient’s autonomy must be respected and she should have a vaginal delivery if she so chooses.
To contact either psychiatry to evaluate the patient (choice A) or the hospital lawyers to get a court order for cesarean delivery (choice B) would be incorrect. Patient autonomy must be respected when it comes to the decision of whether to have a cesarean delivery. This patient has weighed the benefits and risks and has put a great deal of deliberation into her decision. There is no need to involve the psychiatry department or the hospital lawyers in this decision.
To perform cesarean delivery at 38 weeks (choice C) or once the patient is in labor (choice D) would not be correct. In HIV-positive women who do want an elective cesarean delivery, the delivery should be performed at 38 weeks to avoid the risk of labor or rupture of membranes. Once labor starts or the membranes are ruptured, the risk of HIV transmission increases. However, this patient does not want a cesarean delivery, so that operation should not be performed. -
Question 66 of 151
66. Question
19-year-old student is admitted to the psychiatry inpatient unit with the working diagnosis of first-break psychosis. Because of combativeness and threats to others, he is put in seclusion. At first, he refused to take medication by mouth; however, after attacking a nurse he is given haloperidol intramuscularly on two occasions. He has now developed acute torticollis and twitching of the mouth and face on that side. The family is furious, stating that the treatment caused the seizures. Which of the following reactions did this patient most likely have?
Correct
The correct answer is A. Acute dystonia is an involuntary spasm of a particular group of muscles that can involve the neck, jaw, tongue, eyes, or the entire body. It can be an early adverse effect of antipsychotics, and it is more common in younger men. It is more common with typical antipsychotics. The treatment of choice is parenteral administration of anticholinergics.
Akathisia (choice B) is a subjective feeling of muscle discomfort and restlessness that can cause agitation, pacing, anxiety, and dysphoria. It is related to the use of antipsychotics and can appear any time during treatment. It is treated by antipsychotic dose reduction, propranolol, or benzodiazepines. Asterixis (choice C) consists of coarse arrhythmic lapses of sustained posture. It is related to metabolic disorders and is most easily seen when the patient’s arms are outstretched. Asterixis occurs bilaterally.
Lennox-Gastaut syndrome (choice D) is a variant of petit mal epilepsy and consists of intellectual impairment, distinctive slow spike and wave pattern, and atonic postural lapses followed by minor tonic-clonic spasms.
Pseudoseizures (choice E) are part of conversion disorder and are not directly related to the use of antipsychotics. They are very much like real seizures except that there is no aura and no EEG abnormalities. The movements are asynchronous and non-stereotyped, and they occur when the person is awake.Incorrect
The correct answer is A. Acute dystonia is an involuntary spasm of a particular group of muscles that can involve the neck, jaw, tongue, eyes, or the entire body. It can be an early adverse effect of antipsychotics, and it is more common in younger men. It is more common with typical antipsychotics. The treatment of choice is parenteral administration of anticholinergics.
Akathisia (choice B) is a subjective feeling of muscle discomfort and restlessness that can cause agitation, pacing, anxiety, and dysphoria. It is related to the use of antipsychotics and can appear any time during treatment. It is treated by antipsychotic dose reduction, propranolol, or benzodiazepines. Asterixis (choice C) consists of coarse arrhythmic lapses of sustained posture. It is related to metabolic disorders and is most easily seen when the patient’s arms are outstretched. Asterixis occurs bilaterally.
Lennox-Gastaut syndrome (choice D) is a variant of petit mal epilepsy and consists of intellectual impairment, distinctive slow spike and wave pattern, and atonic postural lapses followed by minor tonic-clonic spasms.
Pseudoseizures (choice E) are part of conversion disorder and are not directly related to the use of antipsychotics. They are very much like real seizures except that there is no aura and no EEG abnormalities. The movements are asynchronous and non-stereotyped, and they occur when the person is awake. -
Question 67 of 151
67. Question
31-year-old, HIV-positive woman, gravida 3, para 2, at 32-weeks’ gestation comes to the physician for a prenatal visit. Her prenatal course is significant for the fact that she has taken zidovudine throughout the pregnancy. Otherwise, her prenatal course has been unremarkable. She has no history of mental illness. She states that she has been weighing the benefits and risks of cesarean delivery in preventing transmission of the virus to her baby. After much deliberation, she has decided that she does not want a cesarean delivery and would like to attempt a vaginal delivery. Which of the following is the most appropriate next step in management?
Correct
The correct answer is E. Cesarean delivery has been shown to decrease the rate of transmission of HIV from an infected mother to her fetus. Some reports have shown that the transmission rate can be decreased to as low as 2% with the combination of antiretroviral medication and elective cesarean delivery prior to labor or rupture of membranes. However, although cesarean delivery benefits the infant by decreasing the risk of transmission, the risks of the surgery accrue to the mother. Risks of cesarean delivery include bleeding, infection, and injury to internal organs. HIV-infected women with low CD4 cell counts are known to have higher rates of postoperative complications. Thus, the decision of whether to have a cesarean ultimately belongs to the mother. This patient’s autonomy must be respected and she should have a vaginal delivery if she so chooses.
To contact either psychiatry to evaluate the patient (choice A) or the hospital lawyers to get a court order for cesarean delivery (choice B) would be incorrect. Patient autonomy must be respected when it comes to the decision of whether to have a cesarean delivery. This patient has weighed the benefits and risks and has put a great deal of deliberation into her decision. There is no need to involve the psychiatry department or the hospital lawyers in this decision.
To perform cesarean delivery at 38 weeks (choice C) or once the patient is in labor (choice D) would not be correct. In HIV-positive women who do want an elective cesarean delivery, the delivery should be performed at 38 weeks to avoid the risk of labor or rupture of membranes. Once labor starts or the membranes are ruptured, the risk of HIV transmission increases. However, this patient does not want a cesarean delivery, so that operation should not be performed.Incorrect
The correct answer is E. Cesarean delivery has been shown to decrease the rate of transmission of HIV from an infected mother to her fetus. Some reports have shown that the transmission rate can be decreased to as low as 2% with the combination of antiretroviral medication and elective cesarean delivery prior to labor or rupture of membranes. However, although cesarean delivery benefits the infant by decreasing the risk of transmission, the risks of the surgery accrue to the mother. Risks of cesarean delivery include bleeding, infection, and injury to internal organs. HIV-infected women with low CD4 cell counts are known to have higher rates of postoperative complications. Thus, the decision of whether to have a cesarean ultimately belongs to the mother. This patient’s autonomy must be respected and she should have a vaginal delivery if she so chooses.
To contact either psychiatry to evaluate the patient (choice A) or the hospital lawyers to get a court order for cesarean delivery (choice B) would be incorrect. Patient autonomy must be respected when it comes to the decision of whether to have a cesarean delivery. This patient has weighed the benefits and risks and has put a great deal of deliberation into her decision. There is no need to involve the psychiatry department or the hospital lawyers in this decision.
To perform cesarean delivery at 38 weeks (choice C) or once the patient is in labor (choice D) would not be correct. In HIV-positive women who do want an elective cesarean delivery, the delivery should be performed at 38 weeks to avoid the risk of labor or rupture of membranes. Once labor starts or the membranes are ruptured, the risk of HIV transmission increases. However, this patient does not want a cesarean delivery, so that operation should not be performed. -
Question 68 of 151
68. Question
A 55-year-old woman falls in the shower and hurts her right shoulder. She shows up in the emergency department with her arm held close to her body, but the forearm rotated outward as if she were going to shake hands. She is in pain and will not move the arm from that position. Her shoulder looks “square” in comparison with the rounded unhurt opposite side, and there is numbness in a small area of her shoulder over the deltoid muscle. Which of the following is the most likely diagnosis?
Correct
The correct answer is B. Anterior dislocation of the shoulder is the most common dislocation of that joint. The position is classic, as is the lack of the rounded contour of the humeral head. The area of numbness represents injury to the axillary nerve, a common complication of anterior dislocation of the shoulder.
Acromioclavicular separation (choice A) would be characterized by very localized pain at that particular spot and none of the other features described here.
Fracture of the humeral shaft (choice C) would likewise lack the specific deformity, inasmuch as the humeral head would still be in place to provide the normal rounded contour. Posterior dislocation (choice D) typically occurs following massive uncoordinated muscle contractions (electrical injuries, epileptic seizures), and the arm and forearm are held in a more “normal” protective position, close to the body.
Scapular fracture (choice E) happens only with extremely severe chest trauma; it would not happen by falling in the shower. Along with two other injuries (fracture of the sternum or fracture of the first rib), scapular fracture, when present, indicates that very severe trauma has occurred, and it is a useful clinical clue to look for hidden internal injuries.Incorrect
The correct answer is B. Anterior dislocation of the shoulder is the most common dislocation of that joint. The position is classic, as is the lack of the rounded contour of the humeral head. The area of numbness represents injury to the axillary nerve, a common complication of anterior dislocation of the shoulder.
Acromioclavicular separation (choice A) would be characterized by very localized pain at that particular spot and none of the other features described here.
Fracture of the humeral shaft (choice C) would likewise lack the specific deformity, inasmuch as the humeral head would still be in place to provide the normal rounded contour. Posterior dislocation (choice D) typically occurs following massive uncoordinated muscle contractions (electrical injuries, epileptic seizures), and the arm and forearm are held in a more “normal” protective position, close to the body.
Scapular fracture (choice E) happens only with extremely severe chest trauma; it would not happen by falling in the shower. Along with two other injuries (fracture of the sternum or fracture of the first rib), scapular fracture, when present, indicates that very severe trauma has occurred, and it is a useful clinical clue to look for hidden internal injuries. -
Question 69 of 151
69. Question
37-year-old man lives alone and has no close friends. He works during the night shift at the post office and has little interaction with others. He has not engaged in sexual activity since he was 18 years old, but he does not feel much desire. He maintains a close relationship with his sister, but does not seek out relationships with others. People have told him that he seems “detached” and that he has difficulty experiencing or expressing emotions. Which of the following is the most likely diagnosis?
Correct
Incorrect
The correct answer is D. This patient has schizoid personality disorder, which is characterized by the inability to form relationships and difficulty experiencing and expressing emotions. Affected individuals do not seek intimacy and approval from others. They prefer to be alone and may perform well in socially isolated jobs. The incidence is thought to be very high, however it is not known since these individuals generally do not seek help. According to the DSM, personality disorders are characterized by a stable pattern of behavior that deviates from cultural expectations, is inflexible, cause distress, and social or work impairment.
Personality disorders are not caused by another medical illness or substance abuse. Individuals with antisocial personality disorder (choice A) show a complete disregard for societal norms, rules, and the interest of others. They frequently end up in prison.
Avoidant personality disorder (choice B) is characterized by social withdrawal due to the fear of criticism. Affected individuals are eager to please and desire affection.
Individuals with paranoid personality disorder (choice C) are suspicious and mistrustful, and frequently misinterpret the actions of others.
Schizotypal personality disorder (choice E) is characterized by odd behavior, cognition, and perception. These people seem similar to schizophrenics, but do not have psychosis.
Socially phobic (choice F) individuals desire relationships but their anxiety often interferes with the formation of friendships. They fear unfamiliar situations and scrutiny by others. -
Question 70 of 151
70. Question
A 45-year-old chronic alcoholic presented to the emergency department 5 years ago with 24 hours of epigastric pain radiating to his back, nausea, and vomiting. He gradually recovered from this acute episode. Over the next 5 years, he is repeatedly admitted for similar symptoms. He then presents with gradual onset of weight loss, midabdominal pain radiating to his back, and steatorrhea. Which is of the following has most likely occurred?
Correct
The correct answer is B. Severe epigastric pain radiating to the back and accompanied by nausea and vomiting suggests pancreatitis. Pancreatitis is most likely to be encountered in alcoholics (such as this man) and patients with biliary tract disease. This patient has a history of recurrent alcoholic pancreatitis. The development of gradual weight loss, chronic pain radiating to the back, and steatorrhea suggests that he has now developed chronic pancreatitis. This condition is often complicated by both endocrine and exocrine insufficiency of the pancreas, secondary to loss of much of the tissue of the pancreas to the disease process. Cholangiocarcinoma (choice A) produces jaundice, has a very poor prognosis, and usually occurs in patients with a prior history of primary sclerosing cholangitis.
Gastric outlet obstruction (choice C) would present with postprandial vomiting and early satiety.
Pancreatic adenocarcinoma (choice D) may also occur in this patient but is generally present in older individuals.
There is no evidence of scarring of the common bile duct (choice E), which would produce jaundice.Incorrect
The correct answer is B. Severe epigastric pain radiating to the back and accompanied by nausea and vomiting suggests pancreatitis. Pancreatitis is most likely to be encountered in alcoholics (such as this man) and patients with biliary tract disease. This patient has a history of recurrent alcoholic pancreatitis. The development of gradual weight loss, chronic pain radiating to the back, and steatorrhea suggests that he has now developed chronic pancreatitis. This condition is often complicated by both endocrine and exocrine insufficiency of the pancreas, secondary to loss of much of the tissue of the pancreas to the disease process. Cholangiocarcinoma (choice A) produces jaundice, has a very poor prognosis, and usually occurs in patients with a prior history of primary sclerosing cholangitis.
Gastric outlet obstruction (choice C) would present with postprandial vomiting and early satiety.
Pancreatic adenocarcinoma (choice D) may also occur in this patient but is generally present in older individuals.
There is no evidence of scarring of the common bile duct (choice E), which would produce jaundice. -
Question 71 of 151
71. Question
7-year-old boy is brought to the clinic for a lifetime history of bedwetting. He has otherwise been completely healthy and has met all development milestones. His parents deny a history of trauma, and the history is not consistent with abuse. The patient has been wetting every night but not during the daytime. He has no incontinence. Which of the following is the most appropriate next step in his evaluation?
Correct
The correct answer is D. Given the fact that this patient has had a lifelong history of bed wetting, the initial evaluation will include a urinalysis to rule out infection or bleeding. No neurologic dysfunction exists in this case. An intravenous pyelogram would be needed to evaluate for renal failure or chronic urinary tract infections (choice A).
A renal ultrasound would help evaluate structural damage but is not indicated in the initial evaluation (choice B). 24-hour urine collection is commonly done in the evaluation of nephropathy in diabetics (choice C).
A CT scan at this time would not be indicated at this stage of evaluation (choice E).Incorrect
The correct answer is D. Given the fact that this patient has had a lifelong history of bed wetting, the initial evaluation will include a urinalysis to rule out infection or bleeding. No neurologic dysfunction exists in this case. An intravenous pyelogram would be needed to evaluate for renal failure or chronic urinary tract infections (choice A).
A renal ultrasound would help evaluate structural damage but is not indicated in the initial evaluation (choice B). 24-hour urine collection is commonly done in the evaluation of nephropathy in diabetics (choice C).
A CT scan at this time would not be indicated at this stage of evaluation (choice E). -
Question 72 of 151
72. Question
A 65-year-old man comes to medical attention because of multiple neurologic deficits, including loss of sensation in his right hand, weakness of the left lower leg, and a visual field deficit. He has smoked two packs of cigarettes daily for 40 years. His medical history is remarkable for asthma and emphysema. Examination of the fundus is unremarkable. His temperature is 37 C (98.6 F), blood pressure is 137/86 mm Hg, pulse is 86/min, and respirations are 24/min. MRI of the head reveals five different intracerebral lesions, ranging from 1 to 3 cm in main diameter and located at the gray-white matter junction in both cerebral hemispheres. The lesions are sharply demarcated, and contrast enhancement after gadolinium administration is present in all of them. Which of the following is the most likely diagnosis?
Correct
The correct answer is D. Awareness of the typical MRI/CT appearance of brain metastases is important because often patients present with cerebral metastases without prior history of cancer disease. Thus, a radiologic diagnosis of brain metastatic disease may prompt a search for the underlying primary tumor, which is often a lung carcinoma in men and lung or breast carcinoma in woman. Melanomas also have a peculiar propensity to metastasize to the brain. The typical radiologic features of brain metastases are summarized in this case: multiplicity of lesions, well-circumscribed borders, and location at the gray-white matter junction.
Multiple abscesses (choice A) may develop in patients with sepsis and, particularly, in association with conditions leading to septic emboli (e.g., infective endocarditis).
Arteriovenous malformations (AVMs) (choice B) are abnormal conglomerates of disorganized blood vessels composed of arteries, veins, and intermediate vessels with discontinuous elastic lamina. Intracerebral hemorrhage is their most frequent mode of clinical presentation.
Embolic infarcts (choice C) would be associated with wedge-shaped cortical lesions.
Frequently, embolic infarcts are hemorrhagic. Multifocal glioblastoma multiforme (GBM) (choice E), the most frequent malignant primary brain neoplasm, manifests as an ill-defined mass in the white matter. Contrast enhancement is usually present. Multifocal GBM is a rare event. Even so, the lesions are poorly circumscribed and centered in the white matter.Incorrect
The correct answer is D. Awareness of the typical MRI/CT appearance of brain metastases is important because often patients present with cerebral metastases without prior history of cancer disease. Thus, a radiologic diagnosis of brain metastatic disease may prompt a search for the underlying primary tumor, which is often a lung carcinoma in men and lung or breast carcinoma in woman. Melanomas also have a peculiar propensity to metastasize to the brain. The typical radiologic features of brain metastases are summarized in this case: multiplicity of lesions, well-circumscribed borders, and location at the gray-white matter junction.
Multiple abscesses (choice A) may develop in patients with sepsis and, particularly, in association with conditions leading to septic emboli (e.g., infective endocarditis).
Arteriovenous malformations (AVMs) (choice B) are abnormal conglomerates of disorganized blood vessels composed of arteries, veins, and intermediate vessels with discontinuous elastic lamina. Intracerebral hemorrhage is their most frequent mode of clinical presentation.
Embolic infarcts (choice C) would be associated with wedge-shaped cortical lesions.
Frequently, embolic infarcts are hemorrhagic. Multifocal glioblastoma multiforme (GBM) (choice E), the most frequent malignant primary brain neoplasm, manifests as an ill-defined mass in the white matter. Contrast enhancement is usually present. Multifocal GBM is a rare event. Even so, the lesions are poorly circumscribed and centered in the white matter.